Как решать неравенства с модулем: Решение неравенств с модулем

Содержание

25. Неравенства с модулем | Контрольные работы по математике и другим п

I тип: Неравенство содержит некоторое выражение под модулем и число вне модуля:

где (3.27)

Решение зависит от знака числа А.

1. Если то неравенство (3.27) не имеет решений.

2. Если то неравенство (3.27) равносильно системе неравенств

где (3.28)

1. Если то неравенство (3.28) не имеет решений.

2. Если то неравенство (3.28) равносильно уравнению

3. Если , то неравенство (3.28) равносильно системе неравенств

где (3.29)

1. Если то решением неравенства (3.29) является множество всех значений Х из ОДЗ выражения

2. Если то решением неравенства (3.29) является множество всех значений Х из ОДЗ выражения таких, что

3. Если то неравенство (3.29) равносильно совокупности

где (3.30)

1. Если то решением неравенства (3.30) является множество всех значений Х из ОДЗ выражения

2. Если то неравенство (3.30) равносильно совокупности

II тип: Неравенство, которое содержит выражение с переменной под знаком модуля и вне его:

(3.31)

Где – некоторые выражения с переменной Х.

Для решения неравенств типа (3.31) можно использовать следующие способы.

1-й способ: используя определение модуля, получаем равносильную совокупность систем:

2-й способ: Решаем аналогично решению неравенства (3.29) при дополнительном ограничении на знак выражения

1. Если

(3.32)

То решением является множество всех значений Х из ОДЗ выражения которые удовлетворяют условию (3.32).

2. Если

То решением является множество всех значений Х, которые удовлетворяют системе

3. Если решение определяется системой

Ответом в решении неравенства (3.31) является объединение всех решений, полученных на этапах 1–3.

3-й способ: метод интервалов.

Для решения необходимо:

1) найти значения Х, для которых

2) найденные значения Х нанести на числовую ось;

3) определить знак выражения на всех полученных промежутках;

4) нарисовать кривую знаков;

5) раскрыть модуль, пользуясь рисунком, и получить соответствующее неравенство, которое следует решить вместе с условием принадлежности переменной Х определенному промежутку;

6) в ответе неравенства указать совокупность полученных решений.

III тип: Неравенство содержит несколько модулей и решается двумя способами:

1-й способ: Можно использовать определение модуля и решать совокупность систем неравенств. Этот способ, как правило, не является рациональным.

2-й способ: использовать метод интервалов. Необходимо нарисовать столько числовых осей и кривых знаков, сколько модулей содержится в неравенстве. Для каждого промежутка следует решать полученное после раскрытия модулей неравенство при условии, что переменная Х принадлежит конкретному промежутку. В ответе указывают объединение всех полученных решений.

IV тип: Неравенство вида

где (3.33)

Решается двумя способами:

1-й способ: метод интервалов.

2-й способ: согласно теореме равносильности (см. свойства равносильности неравенств (3.22) и (3.23)) неравенство (3.33) можно возводить в квадрат:

Решение неравенства (3.33) сводится к решению неравенства

Аналогично решают неравенства IV типа (3.33), если они заданы со знаками

V тип: Неравенства, решаемые заменой переменной.

В таком случае выражение с модулем обозначают новой переменной. Неравенство с новой переменной решают до конца (т. е. до возможного получения промежутков решения для новой переменной). Затем возвращаются к старой переменной и решают полученные неравенства с модулем как неравенства I типа.

Пример 1. Решить неравенства:

1) 2)

3) 4)

5) 6)

Решение. 1) Решаем как неравенство I типа:

Получаем ответ:

2) Решаем как неравенство I типа:

Второе неравенство совокупности не имеет решения (соответствующая парабола лежит над осью Ох). Первое неравенство сводится к виду

Его решение: это и есть ответ.

3) Решаем как неравенство II типа. Оно имеет решение, если Поэтому получаем равносильную систему:

Получаем ответ:

4) Заданное неравенство может быть записано в виде

Заменим переменную Решаем неравенство

Его решение

Возвращаемся к переменной Х и решаем совокупность

Получаем

Т. е. приходим к ответу

5) Для решения неравенства используем метод интервалов. Запишем неравенство в виде

Построим числовые прямые и определим знаки выражений, стоящих под модулем (рис. 3.10).

ОДЗ:

Рис. 3.10

А) рассмотрим неравенство на 1-м промежутке. Получаем систему

(3.34)

Решаем неравенство

Получаем

Система (3.34) сводится к системе

На данном промежутке решений нет.

Б)

Если , то С учетом рассматриваемого промежутка имеем:

Получаем

В)

Решением является промежуток:

Объединим полученные решения и приходим к ответу:

6)

ОДЗ:

Введем новую переменную:

тогда и приходим к неравенству вида

Решаем его

Используем метод интервалов (рис. 3.11).

Рис. 3.11

Запишем полученное решение в виде совокупности:

Вернемся к переменной Х:

(3.35)

– выполняется при любых

С учетом ОДЗ второе неравенство системы (3.35) равносильно системе

Получаем ответ:

< Предыдущая   Следующая >

Иррациональные неравенства. Неравенства с модулем

Дата публикации: .

Ребята, на данном уроке мы рассмотрим способы решений двух видов неравенств. Они могут пригодиться при подготовке к единому государственному экзамену, если вы будете решать задачи из второй части экзамена.

Рассмотрим неравенство вида: $\sqrt{f(x)}<g(x)$.
Такие неравенства называются иррациональными. В нашем выражении присутствует корень квадратный, который накладывает свои ограничения на область допустимых значений.

Первое, что мы должны иметь в виду, корень квадратный извлекается только из положительных чисел, поэтому $f(x)≥0$.2 \end {cases}$.

Решим систему: $\begin {cases} (x-5)(x+6)≥0, \\ x>0, \\ x

Воспользуемся методом интервалов.

Решения всех трех неравенств системы пересекаются на отрезке $[5;30)$, который и есть решение исходного неравенства.
б) Нам нужно решить совокупность двух систем:

Решения каждой системы очевидны, даже не применяя метод интервалов:

Ответ: а) $хϵ[5;30)$: б) $хϵ(-∞;6]U(30;+∞)$.

Неравенства с модулями

Неравенства, которые содержат переменную под знаком модуля, могут решаться разными методами. Мы рассмотрим три способа решения. Советую применять третий способ. Он может занимать немного больше времени, но зато гораздо больше шансов на правильное решение неравенства.

Пример.
Решить неравенство: $|3x-6|>6$.

Решение.
Первый способ.
$|3(x-2)|>6$; $|(x-2)|>2$.
Как мы помним, геометрически смысл модуля есть ни что иное, как расстояние между точка х и 2. Согласно нашему неравенству оно должно превышать 2. Тогда решением будет $(-∞;0)U(4;+∞)$.2$.

Метод интервалов – универсальный метод решения неравенств с модулем

Чем больше человек понимает, тем сильнее в нем желание понимать

Фома Аквинский

Метод интервалов позволяет решать любые уравнения, содержащие модуль. Суть этого метода в том, чтобы разбить числовую ось на несколько участков (интервалов), причем разбить ось нужно именно нулями выражений, стоящих в модулях. Затем на каждом из получившихся участков всякое подмодульное выражение либо положительно, либо отрицательно. Поэтому каждый из модулей может быть раскрыт или со знаком минус, или со знаком плюс. После этих действий остается лишь решить каждое из полученных простых уравнений на рассматриваемом интервале и объединить полученные ответы.

Рассмотрим данный метод на конкретном примере.

|x + 1| + |2x – 4| – |x + 3| = 2x – 6.

1) Найдем нули выражений, стоящих в модулях. Для этого нужно приравняем их к нулю, и решить полученные уравнения.

x + 1 = 0    2x – 4 = 0     x + 3 = 0

x = -1         2x = 4           x = -3

                  x = 2

2) Расставим получившиеся точки в нужном порядке на координатной прямой. Они разобьют всю ось на четыре участка.

3) Определим на каждом из получившихся участков знаки выражений, стоящих в модулях. Для этого подставляем в них любые числа с интересующих нас интервалов. Если результат вычислений – число положительное, то в таблице ставим «+», а если число отрицательное, то ставим «–». Это можно изобразить так:

4) Теперь будем решать уравнение на каждом из четырех интервалов, раскрывая модули с теми знаками, которые проставлены в таблице. Итак, рассмотрим первый интервал:

I интервал (-∞; -3). На нем все модули раскрываются со знаком «–». Получим следующее уравнение:

-(x + 1) – (2x – 4) – (-(x + 3)) = 2x – 6. Приведем подобные слагаемые, раскрыв предварительно скобки в полученном уравнении:

-x – 1 – 2x + 4 + x + 3 = 2x – 6

-4x = -12

x = 3.

Полученный ответ не входит в рассматриваемый интервал, поэтому в окончательный ответ писать его не надо.

II интервал [-3; -1). На этом интервале в таблице стоят знаки «–», «–», «+». Именно так и раскрываем модули исходного уравнения:

-(x + 1) – (2x – 4) – (x + 3) = 2x – 6. Упростим, раскрыв при этом скобки:

-x – 1 – 2x + 4 – x – 3 = 2x – 6. Приведем в полученном уравнении подобные:

-5x = -6

x = 6/5. Полученное число не принадлежит рассматриваемому интервалу, поэтому оно не является корнем исходного уравнения.

III интервал [-1; 2). Раскрываем модули исходного уравнения с теми знаками, которые стоят на рисунке в третьей колонке. Получаем:

(x + 1) – (2x – 4) – (x + 3) = 2x – 6. Избавимся от скобок, перенесем слагаемые, содержащие переменную x в левую часть уравнения, а не содержащие x в правую. Будем иметь:

x + 1 – 2x + 4 – x – 3 = 2x – 6

-4x = -8

x = 2.

В рассматриваемый интервал число 2 не входит.

IV интервал [2; +∞). Все модули раскрываем со знаком «+». Получим:

(x + 1) + (2x – 4) – (x + 3) = 2x – 6.

x + 1 + 2x – 4 – x – 3 = 2x – 6

0 = 0.

После преобразований уравнение превратилось в верное равенство. Это говорит о том, что любое число из рассматриваемого интервала будет являться решением исходного уравнения. Значит ответом,  как на этом интервале, так и во всем уравнении является множество чисел, удовлетворяющих условию x ≥ 2.

Ответ: x ≥ 2.

Метод интервалов хоть и является универсальным методом решения уравнений с модулем, его применение не всегда оправдано. Порой решить уравнение выходит гораздо быстрее, используя, например, определение модуля или какие-то другие методы.

© blog.tutoronline.ru,
при полном или частичном копировании материала ссылка на первоисточник обязательна.

3.2.4. Неравенства с модулем

Основные
способы решений неравенств с модулем
во многом совпадают с методами решения
аналогичных уравнений. Единственное
отличие, пожалуй, связано с тем, что,
решая неравенства с модулем (как, впрочем,
и неравенства вообще), нужно очень
внимательно совершать равносильные
переходы и следить не только за тем,
чтобы не приобрести новые решения, но
и за тем, чтобы не потерять уже имеющиеся.

Стандартный
путь решения неравенств с модулем
заключается в том, что координатная
прямая разбивается на промежутки
(границами этих промежутков являются
нули подмодульных выражений), а затем
неравенство решается на каждом из
промежутков.

Этот
метод работает всегда. Правда, в отдельных
случаях может быть затруднена его
техническая реализация, например, очень
тяжело или невозможно найти корни
подмодульных выражений и пр. Однако,
это сложности иного плана. Нужно понимать,
что раскрытие модуля по определению
неизменно приводит к цели. Конечно же,
этот метод не является оптимальным: в
условиях конкурсного экзамена важен
не только результат, но и то время,
которое потрачено на его получение.

Рассмотрим
методы, не связанные с поиском нулей
функций, стоящих под знаком модуля.

Рассмотрим
неравенство

Очевидно,
что те x, для которых g (x) < 0, не
являются решениями. Значит, если x
является решением, то для него g (x) ≥ 0,
и согласно геометрическому смыслу
модуля, как расстоянию на координатной
оси, данное неравенство равносильно
системе

Таким
образом, имеем

Аналогично
можно рассмотреть неравенство

Неравенство
выполнено для тех x, для которых g (x) < 0
и функции f (x) и g (x) определены. Для
тех x, для которых g (x) ≥ 0, имеем
равносильную совокупность

Заметим,
что последняя совокупность является
равносильной нашему неравенству и при
g (x) ≤ 0. В этом можно непосредственно
убедиться, учтя g (x) ≤ 0 и вспомнив
определение знака совокупности.

Пример 1

Решите
неравенство

Решение

Перейдём
к равносильной совокупности.

Ответ. 

Как
видно, в простых случаях особых преимуществ
метод перехода к равносильной системе
не имеет, но иногда его преимущества
весьма заметны.

Пример 2

Решите
неравенство

Решение

Как
видно, найти значения x, при которых
подмодульное выражение обращается в
нуль, чрезвычайно затруднительно.
Однако переход к равносильной системе
значительно упрощает дело. Имеем:

Ответ. 

3.2.5. Тригонометрические неравенства

При
решении тригонометрических неравенств
вида f (x) ≥ 0, где f (x) − одна
из тригонометрических функций, удобно
использовать тригонометрическую
окружность для того, чтобы наиболее
наглядно представить решения неравенства
и записать ответ. Основным методом
решения тригонометрических неравенств
является сведение их к простейшим
неравенствам типа

Разберём
на примере, как решать такие неравенства.

Пример 1

Решите
неравенство

Решение

Для
решения неравенств с тангенсом и
котангенсом полезно понятие о линии
тангенсов и котангенсов. Таковыми
являются прямые x = 1 и y = 1
соответственно, касающиеся тригонометрической
окружности.

Рисунок
3.2.5.1

Легко
заметить, что если построить луч с
началом в начале координат, составляющий
угол α с положительным направлением
оси абсцисс, то длина отрезка от точки
(1; 0) до точки пересечения этого луча
с линией тангенсов в точности равна
тангенсу угла, который составляет этот
луч с осью абсцисс. Аналогичное наблюдение
имеет место и для котангенса.

Пример 2

Решите
неравенство

Решение

Пример 3

Решите
неравенство

Решение

Комплексные неравенства с модулем. Калькулятор онлайн.Решение уравнений и неравенств с модулями

решение неравенства
в режиме онлайн
решение
почти любого заданного неравенства онлайн
. Математические неравенства онлайн
для решения математики. Быстро найти решение неравенства
в режиме онлайн
. Сайт www.сайт позволяет найти решение
почти любого заданного алгебраического
, тригонометрического
или трансцендентного неравенства онлайн
. При изучении практически любого раздела математики на разных этапах приходится решать неравенства онлайн
. Чтобы получить ответ сразу, а главное точный ответ, необходим ресурс, позволяющий это сделать. Благодаря сайту www.сайт решение неравенства онлайн
займет несколько минут. Основное преимущество www.сайт при решении математических неравенства онлайн
— это скорость и точность выдаваемого ответа. Сайт способен решать любые алгебраические неравенства онлайн
, тригонометрические неравенства онлайн
, трансцендентные неравенства онлайн
, а также неравенства
с неизвестными параметрами в режиме онлайн
. Неравенства
служат мощным математическим аппаратом решения
практических задач. C помощью математических неравенств
можно выразить факты и соотношения, которые могут показаться на первый взгляд запутанными и сложными. Неизвестные величины неравенств
можно найти, сформулировав задачу на математическом
языке в виде неравенств
и решить
полученную задачу в режиме онлайн
на сайте www.сайт. Любое алгебраическое неравенство
, тригонометрическое неравенство
или неравенства
содержащие трансцендентные
функции Вы легко решите
онлайн и получите точный ответ. Изучая естественные науки, неизбежно сталкиваешься с необходимостью решения неравенств
. При этом ответ должен быть точным и получить его необходимо сразу в режиме онлайн
. Поэтому для решения математических неравенств онлайн
мы рекомендуем сайт www.сайт, который станет вашим незаменимым калькулятором для решения алгебраических неравенств онлайн
, тригонометрических неравенств онлайн
, а также трансцендентных неравенств онлайн
или неравенств
с неизвестными параметрами. Для практических задач по нахождению инетравол решений различных математических неравенств
ресурса www.. Решая неравенства онлайн
самостоятельно, полезно проверить полученный ответ, используя онлайн решение неравенств
на сайте www.сайт. Необходимо правильно записать неравенство и моментально получите онлайн решение
, после чего останется только сравнить ответ с Вашим решением неравенства. Проверка ответа займет не более минуты, достаточно решить неравенство онлайн
и сравнить ответы. Это поможет Вам избежать ошибок в решении
и вовремя скорректировать ответ при решении неравенств онлайн
будь то алгебраическое
, тригонометрическое
, трансцендентное
или неравенство
с неизвестными параметрами.

Приложение

Решение неравенств онлайн на Math34.biz для закрепления студентами и школьниками пройденного материала. И тренировки своих практических навыков. Неравенство в математике — утверждение об относительной величине или порядке двух объектов (один из объектов меньше или не больше другого), или о том, что два объекта не одинаковы (отрицание равенства). В элементарной математике изучают числовые неравенства, в общей алгебре, анализе, геометрии рассматриваются неравенства также и между объектами нечисловой природы. Для решения неравенства обязательно должны быть определены обе его части с одним из знаков неравенства между ними. Строгие неравенства подразумевают неравенство двух объектов. В отличие от строгих, нестрогие неравенства допускают равенство входящих в него объектов. Линейные неравенства представляют собой простейшие с точки зрения начала изучения выражения, и для решения таких неравенств используются самые простые методики. Главная ошибка учеников в решении неравенств онлайн в том, что они не различают особенность строгого и нестрогого неравенства, от чего зависит войдут или нет граничные значения в конечный ответ. Несколько неравенств, связанных между собой несколькими неизвестными, называют системой неравенств. Решением неравенств из системы является некая область на плоскости, либо объемная фигура в трехмерном пространстве. Наряду с этим абстрагируются n-мерными пространствами, однако при решении таких неравенств зачастую не обойтись без специальных вычислительных машин. Для каждого неравенства в отдельности нужно найти значения неизвестного на границах области решения. Множество всех решений неравенства и является его ответом. Замена одного неравенства равносильным ему другим неравенством называется равносильным переходом от одного неравенства к другому. Аналогичный подход встречается и в других дисциплинах, потому что помогает привести выражения к стандартному виду. Вы оцените по достоинству все преимущества решение неравенств онлайн на нашем сайте. Неравенство — это выражение, содержащее один из знаков = >. По сути это логическое выражение. Оно может быть либо верным, либо нет — в зависимости от того, что стоит справа и слева в этом неравенстве. Разъяснение смысла неравенства и основные приемы решения неравенств изучаются на разных курсах, а также в школе. Решение любых неравенств онлайн — неравенства с модулем, алгебраические, тригонометрические, трансцендентные неравенства онлайн. Тождественное неравенство, как строгие и нестрогие неравенства, упрощают процесс достижения конечного результата, являются вспомогательным инструментом для разрешения поставленной задачи. Решение любых неравенств и систем неравенств, будь то логарифмические, показательные, тригонометрические или квадратных неравенства, обеспечивается с помощью изначально правильного подхода к этому важному процессу. Решение неравенств онлайн на сайте сайт всегда доступно всем пользователям и абсолютно бесплатно. Решениями неравенства с одной переменной называются значения переменной, которые обращают его в верное числовое выражение. Уравнения и неравенства с модулем: модуль действительного числа — это абсолютная величина этого числа. Стандартный метод решения этих неравенств заключается в возведении обеих частей неравенства в нужную степень. Неравенства – это выражения, указывающие на сравнение чисел, поэтому грамотное решение неравенств обеспечивает точность таких сравнений. Они бывают строгими (больше, меньше) и нестрогими (больше или равно, меньше или равно). Решить неравенство – значит найти все те значения переменных, которые при подстановке в исходное выражение обращают его в верное числовое представление.. Понятие неравенства, его сущность и особенности, классификация и разновидности — вот что определяет специфику данного математического раздела. Основные свойства числовых неравенств, применимые ко всем объектам данного класса, обязательно должны быть изучены учениками на начальном этапе ознакомления с данной темой. Неравенства и промежутки числовой прямой очень тесно связаны, когда речь идет о решении неравенств онлайн. Графическое обозначение решения неравенства наглядно показывает суть такого выражения, становится понятно к чему следует стремиться при решении какой-либо поставленной задачи. В основу понятия неравенства входит сравнение двух или нескольких объектов. Неравенства, содержащие переменную, решаются как аналогично составленные уравнения, после чего делается выборка интервалов, которые будут приняты за ответ. Любое алгебраическое неравенство, тригонометрическое неравенство или неравенства содержащие трансцендентные функции, вы с легкостью и мгновенно сможете решить, используя наш бесплатный сервис. Число является решением неравенства, если при подстановке этого числа вместо переменной получаем верное выражение, то есть знак неравенства показывает истинное понятие.. Решение неравенств онлайн на сайт каждый день для полноценного изучения студентами пройденного материала и закрепления своих практических навыков. Зачастую тема неравенства онлайн в математике изучается школьниками после прохождения раздела уравнений. Как и положено применяются все принципы при решении, чтобы определить интервалы решений. Найти в аналитическом виде ответ бывает сложнее, чем сделать то же самое, но в числовом виде. Однако такой подход дает более наглядное и полное представление об целостности решения неравенства. Сложность может возникнуть на этапе построения линии абсцисс и нанесения точек решения однотипного уравнения. После этого решение неравенств сводится к определению знака функции на каждом выявленном интервале с целью определения возрастания или убывания функции. Для этого необходимо поочередно подставлять к значениям, заключенных внутри каждого интервала, в исходную функцию и проверять её значение на положительность или отрицательность. В этом есть суть нахождения всех решений, в том числе интервалов решений. Когда вы сами решите неравенство и увидите все интервалы с решениями, то поймете, насколько применим такой подход для дальнейших действий. Сайт сайт предлагает вам перепроверить свои результаты вычислений с помощью мощного современного калькулятора на этой странице. Вы сможете с легкостью выявить неточности и недочеты в своих расчетах, использую уникальный решебник неравенств. Студенты часто задаются вопросом, где найти такой полезный ресурс? Благодаря инновационному подходу к возможности определения потребностей инженеров, калькулятор создан на базе мощных вычислительных серверов с использованием только новых технологий. По сути решение неравенств онлайн заключается в решении уравнения с вычислением всех возможных корней. Полученные решения отмечаются на прямой, а далее производится стандартная операция по определению значения функции на каждом промежутке. А что же делать, если корни уравнения получаются комплексные, как в этом случае решить неравенство в полной форме, которое бы удовлетворяло всем правилам написания результата? Ответ на этот и многие другие вопросы с легкость даст наш сервис сайт, для которого нет ничего невозможного в решении математических задач онлайн. В пользу вышесказанного добавим следующее: каждый, кто всерьез занимается изучением такой дисциплиной как математика, обязан изучить тему неравенств. Неравенства бывают разных типов и решить неравенство онлайн порой сделать непросто, так как необходимо знать принципы подходов к каждому из них. На этом базируется основа успеха и стабильности. Для примера можно рассмотреть такие типы, как логарифмические неравенства или трансцендентные неравенства. Это вообще особый вид таких, сложных на первый взгляд, задач для студентов, тем более для школьников. Преподаватели институтов уделяют немало времени из подготовки практикантов для достижения профессиональных навыков в работе. К таким же типам отнесем тригонометрические неравенства и обозначим общий подход при решении множества практических примеров из постановочной задачи. В ряде случаев сначала нужно привести все к уравнению, упростить его, разложить на разные множители, короче говоря, привести к вполне наглядному виду. Во все времена человечество стремилось найти оптимальный подход в любых начинаниях. Благодаря современным технологиям, человечество сделало просто огромный прорыв в будущее свое развитие. Инновации все чаще и чаще, день за днем вливаются в нашу жизнь. В основу вычислительной техники легла, разумеется, математика со своим принципами и строгим подходом к делу. сайт представляет собой общий математический ресурс, в котором имеется разработанный калькулятор неравенств и многие другие полезные сервисы. Используйте наш сайт и у вас будет уверенность в правильности решенных задач. Из теории известно, что объекты нечисловой природы также изучаются неравенствами онлайн, только этот подход представляет собой особый способ изучения данного раздела в алгебре, геометрии и других направлениях математики. Решать неравенства можно по-разному, неизменным остается конечная проверка решений и лучше всего это делать прямой подстановкой значений в само неравенство. Во многих случаях полученный ответ очевиден и его легко проверить в уме. Предположим нам задано решить дробное неравенство, в котором присутствуют искомые переменные в знаменателях дробных выражений. Тогда решение неравенств сведется к приведению всех слагаемых к общему знаменателю, предварительно переместив все в левую и правую часть неравенства. Далее нужно решить однородное уравнение, полученное в знаменателе дроби. Эти числовые корни будут точками, не включенными в интервалы общего решения неравенства, или ка их еще называют — проколотые точки, в которых функция обращается в бесконечность, то есть функция не определена, а можно только получить ее предельное значение в данной точке. Решив полученное в числителе уравнение, все точки нанесем на числовую ось. Заштрихуем те точки, в которых числитель дроби обращаемся в ноль. Соответственно все остальные точки оставляем пустыми или проколотыми. Найдем знак дроби на каждом интервале и после этого выпишем окончательный ответ. Если на границах интервала будут заштрихованные точки, то тогда включаем эти значения в решение. Если на границах интервала будут проколотые точки — эти значения в решение не включаем. После того, как решите неравенство, вам потребуется в обязательном порядке проверить полученный результат. Можно это сделать руками, каждое значение из интервалов ответа поочередно подставить в начальное выражение и выявить ошибки. Сайт сайт с легкостью выдаст вам все решения неравенства, и вы сразу сравните полученные вами и калькулятором ответы. Если все-таки ошибка будет иметь место, то на нашем ресурсе решение неравенств онлайн окажется вам очень полезным. Рекомендуем всем студентам вначале приступать не к решению напрямую неравенства, а сначала получить результат на сайт, потому что в дальнейшем будет намного проще самому сделать правильный расчет. В текстовых задачах практически всегда решение сводится к составлению системы неравенств с несколькими неизвестными. Решить неравенство онлайн в считанные секунды поможет наш ресурс. При этом решение будет произведено мощной вычислительной программой с высокой точностью и без всяких погрешностей в конечном ответе. Тем самым вы сможете сэкономить колоссальное количество времени на решении данным калькулятором примеров. В ряде случаев школьники испытывают затруднения, когда на практике или в лабораторных работах встречают логарифмические неравенства, а еще хуже, когда видят перед собой тригонометрические неравенства со сложными дробными выражениями с синусами, косинусами или вообще с обратными тригонометрическими функциями. Как ни крути, но без помощи калькулятора неравенств справиться будет очень сложно и не исключены ошибки на любом этапе решения задачи. Пользуйтесь ресурсом сайт совершенно бесплатно, он доступен каждому пользователю каждый день. Начинать действовать с нашего сервиса-помощника очень хорошая идея, поскольку аналогов существует множество, а по-настоящему качественных сервисов единицы. Мы гарантируем точность вычислений при длительности поиска ответа в несколько секунд. От вас требуется только записать неравенства онлайн, а мы в свою очередь сразу предоставим вам точный результат решения неравенства. Искать подобный ресурс может оказаться бессмысленным занятием, так как вряд ли вы встретите такой же качественный сервис как у нас. Можно обойтись без теории про решение неравенств онлайн, но без качественного и быстрого калькулятора вам не обойтись. Желаем вам успехов в учебе! По-настоящему выбрать оптимальное решение неравенства онлайн зачастую связано с логическим подходом для случайной величины. Если пренебречь малым отклонением замкнутого поля, то вектор нарастающего значения пропорционален наименьшему значению на промежутке убывания линии ординат. Инвариант пропорционален двукратному увеличению отображаемым функциям наряду с исходящим ненулевым вектором. Лучший ответ всегда содержит точность вычислений. Наше решение неравенств примет вид однородной функции последовательно сопряженных числовых подмножеств главного направления. За первый интервал возьмем как раз наихудшее по точности значение нашего представления переменной. Вычислим на максимальное отклонение предыдущее выражение. Будем пользоваться сервисом на усмотрение предложенных вариантов по мере необходимости. Будет ли найдено решение неравенств онлайн с помощью хорошего в своем классе калькулятора — это риторический вопрос, разумеется, студентам такой инструмент пойдет только на пользу и принесет огромный успех в математике. Наложим ограничение на область с множеством, которое сведем к элементам с восприятием импульсов по напряжению. Физические значения таких экстремумов математически описывают возрастание и убывание кусочно-непрерывных функций. На протяжении всего пути ученые находили доказательства существования элементов на разных уровнях изучения. Расположим все последовательно идущие подмножества одного комплексного пространства в один ряд с такими объектами, как шар, куб или цилиндр. Из нашего результата можно сделать однозначный вывод и когда решите неравенство, то на выходе, безусловно, прольется свет на высказанное математическое предположение об интеграции метода на практике. В текущем положении вещей необходимое условие будет также являться и достаточным условием. Критерии неопределенности зачастую вызывают у студентов разногласия по причине недостоверных данных. Это упущение должны взять на себя преподаватели ВУЗов, а также учителя в школах, так как на начальном этапе обучения необходимо это тоже учитывать. Из вышесказанного вывода на взгляд опытных людей можно делать выводы, что решить неравенство онлайн очень сложное задание при вхождении в неравенство неизвестных разного типа данных. Об этом сказано на научной конференции в западном округе, на которой выдвигали самые различные обоснования по поводу научных открытий в области математики и физики, а также молекулярного анализа биологически устроенных систем. В нахождении оптимального решения абсолютно все логарифмические неравенства представляют научную ценность для всего человечества. Исследуем данный подход на предмет логических заключений по ряду несовпадений на высшем уровне понятий о существующем объекте. Логика подсказывает иное, чем видно на первый взгляд неопытному студенту. По причине возникновения масштабных аналогий, будет рационально сначала приравнять отношения к разности предметов исследуемой области, а затем показать на практике наличие общего аналитического результата. Решение неравенств абсолютным образом завязано на применении теории и будет важно для каждого изучить такой необходимый для дальнейших исследований раздел математики. Однако, при решении неравенств вам нужно найти все корни составленного уравнения, а уже затем нанести все точки на ось ординат. Некоторые точки будут проколоты, а остальные войдут в интервалы с общим решением. Начнем изучать раздел математики с азов важнейшей дисциплины школьной программы. Если тригонометрические неравенства являются неотъемлемой частью текстовой задачи, то, как раз применять ресурс для вычисления ответа просто необходимо. Введите левую и правую части неравенства корректно, нажмите на кнопу и получите результат в течение нескольких секунд. Для быстрых и точных математических вычислений с числовыми или символьными коэффициентами перед неизвестными, вам как всегда понадобится универсальный калькулятор неравенств и уравнений, который сможет в считанные секунды предоставить ответ на поставленную вами задачку. Если у вас нет времени на написание целого ряда письменных упражнений, то обоснованность сервиса неоспорима даже невооруженным глазом. Для студентов такой подход является более оптимальным и оправданным с точки зрения экономии материальных ресурсов и времени. Напротив катета лежит угол, а для его измерения необходим циркуль, но вы сможете в любо момент воспользоваться подсказками и решите неравенство не применяя никаких формул приведения. Означает ли это успешное завершение начатого действия? Однозначно ответ будет положительным.

Модулем числа
называется само это число, если оно неотрицательное, или это же число с противоположным знаком, если оно отрицательное.

Например, модулем числа 6 является 6, модулем числа -6 тоже является 6.

То есть под модулем числа понимается абсолютная величина, абсолютное значение этого числа без учета его знака.

Обозначается так: |6|, |х
|, |а
| и т.д.

(Подробнее — в разделе «Модуль числа»).

Уравнения с модулем.

Пример 1

. Решить уравнение
|10
х
— 5| = 15.

Решение
.

В соответствии с правилом, уравнение равносильно совокупности двух уравнений:

10х
— 5 = 15
10х
— 5 = -15

Решаем:

10х
= 15 + 5 = 20
10х
= -15 + 5 = -10

х
= 20: 10
х
= -10: 10

х
= 2
х
= -1

Ответ
: х
1 = 2, х
2 = -1.

Пример 2

. Решить уравнение
|2
х
+ 1| =
х
+ 2.

Решение
.

Поскольку модуль — число неотрицательное, то х
+ 2 ≥ 0. Соответственно:

х
≥ -2.

Составляем два уравнения:

2х
+ 1 = х
+ 2
2х
+ 1 = -(х
+ 2)

Решаем:

2х
+ 1 = х
+ 2
2х
+ 1 = —х
— 2

2х
х
= 2 — 1
2х
+ х
= -2 — 1

х
= 1
х
= -1

Оба числа больше -2. Значит, оба являются корнями уравнения.

Ответ
: х
1 = -1, х
2 = 1.

Пример 3

. Решить уравнение

|х
+ 3| — 1
————— = 4
х
— 1

Решение
.

Уравнение имеет смысл, если знаменатель не равен нулю — значит, если х
≠ 1. Учтем это условие. Наше первое действие простое — не просто освобождаемся от дроби, а преобрахуем ее так, чтобы получить модуль в чистом виде:

|х
+ 3| — 1 = 4 · (х
— 1),

|х
+ 3| — 1 = 4х
— 4,

|х
+ 3| = 4х
— 4 + 1,

|х
+ 3| = 4х
— 3.

Теперь у нас в левой части уравнения только выражение под модулем. Идем дальше.
Модуль числа есть неотрицательное число — то есть он должен быть больше нуля или равен нулю. Соответственно, решаем неравенство:

4х
— 3 ≥ 0

4х
≥ 3

х
≥ 3/4

Таким образом, у нас появилось второе условие: корень уравнения должен быть не меньше 3/4.

В соответствии с правилом, составляем совокупность двух уравнений и решаем их:

х
+ 3 = 4х
— 3
х
+ 3 = -(4х
— 3)

х
+ 3 = 4х
— 3
х
+ 3 = -4х
+ 3

х
— 4х
= -3 — 3
х
+ 4х
= 3 — 3

х
= 2
х
= 0

Мы получили два ответа. Проверим, являются ли они корнями исходного уравнения.

У нас было два условия: корень уравнения не может быть равен 1, и он должен быть не меньше 3/4. То есть х
≠ 1, х
≥ 3/4. Обоим этим условиям соответствует только один из двух полученных ответов — число 2. Значит, только оно и является корнем исходного уравнения.

Ответ
: х
= 2.

Неравенства с модулем.

Пример 1

. Решить неравенство
|
х
— 3|

Решение
.

Правило модуля гласит:

|а
| = а
, если а
≥ 0.

|а
| = —а
, если а

Модуль может иметь и неотрицательное, и отрицательное число. Значит, мы должны рассмотреть оба случая: х
— 3 ≥ 0 и х
— 3

1) При х
— 3 ≥ 0 наше исходное неравенство остается как есть, только без знака модуля:
х
— 3

2) При х
— 3

-(х
— 3)

Раскрыв скобки, получаем:

х
+ 3

Таким образом, от этих двух условий мы пришли к объединению двух систем неравенств:

х
— 3 ≥ 0
х
— 3

х
— 3 —х
+ 3

Решим их:

х
≥ 3
х

х
х
> -1

Итак, у нас в ответе объединение двух множеств:

3 ≤ х
х

Определяем наименьшее и наибольшее значения. Это -1 и 7. При этом х
больше -1, но меньше 7.
Кроме того, х
≥ 3. Значит, решением неравенства является все множество чисел от -1 до 7, исключая эти крайние числа.

Ответ
: -1 х

Или: х
∈ (-1; 7).

Дополнения
.

1) Есть более простой и короткий способ решения нашего неравенства — графический. Для этого надо нарисовать горизонтальную ось (рис.1).

Выражение |х
— 3| х
до точки 3 меньше четырех единиц. Отмечаем на оси число 3 и отсчитываем влево и вправо от от него 4 деления. Слева мы придем к точке -1, справа — к точке 7. Таким образом, точки х
мы просто увидели, не вычисляя их.

При этом, согласно условию неравенства, сами -1 и 7 не включены во множество решений. Таким образом, получаем ответ:

1 х

2) Но есть еще одно решение, которое проще даже графического способа. Для этого наше неравенство надо представить в следующем виде:

4 х
— 3

Ведь так оно и есть по правилу модуля. Неотрицательное число 4 и аналогичное отрицательное число -4 являются границами решения неравенства.

4 + 3 х

1 х

Пример 2

. Решить неравенство
|
х
— 2| ≥ 5

Решение
.

Этот пример существенно отличается от предыдущего. Левая часть больше 5 либо равна 5. С геометрической точки зрения, решением неравенства являются все числа, которые от точки 2 отстоят на расстоянии 5 единиц и больше (рис.2). По графику видно, что это все числа, которые меньше или равны -3 и больше или равны 7. А значит, мы уже получили ответ.

Ответ
: -3 ≥ х
≥ 7.

Попутно решим это же неравенство способом перестановки свободного члена влево и вправо с противоположным знаком:

5 ≥ х
— 2 ≥ 5

5 + 2 ≥ х
≥ 5 + 2

Ответ тот же: -3 ≥ х
≥ 7.

Или: х
∈ [-3; 7]

Пример решен.

Пример 3

. Решить неравенство
6
х
2 — |
х
| — 2 ≤ 0

Решение
.

Число х
может быть и положительным числом, и отрицательным, и нулем. Поэтому нам надо учесть все три обстоятельства. Как вы знаете, они учитываются в двух неравенствах: х
≥ 0 и х
х
≥ 0 мы просто переписываем наше исходное неравенство как есть, только без знака модуля:

6х 2 — х
— 2 ≤ 0.

Теперь о втором случае: если х

6х
2 — (-х
) — 2 ≤ 0.

Раскрываем скобки:

6х
2 + х
— 2 ≤ 0.

Таким образом, мы получили две системы уравнений:

6х
2 — х
— 2 ≤ 0
х
≥ 0

6х
2 + х
— 2 ≤ 0
х

Надо решить неравенства в системах — а это значит, надо найти корни двух квадратных уравнений. Для этого приравняем левые части неравенств к нулю.

Начнем с первого:

6х
2 — х
— 2 = 0.

Как решается квадратное уравнение — см. раздел «Квадратное уравнение». Мы же сразу назовем ответ:

х
1 = -1/2, х 2 = 2/3.

Из первой системы неравенств мы получаем, что решением исходного неравенства является все множество чисел от -1/2 до 2/3. Пишем объединение решений при х
≥ 0:
[-1/2; 2/3].

Теперь решим второе квадратное уравнение:

6х
2 + х
— 2 = 0.

Его корни:

х
1 = -2/3, х
2 = 1/2.

Вывод: при х

Объединим два ответа и получим итоговый ответ: решением является все множество чисел от -2/3 до 2/3, включая и эти крайние числа.

Ответ
: -2/3 ≤ х
≤ 2/3.

Или: х
∈ [-2/3; 2/3].

Существует несколько способов решения неравенств, содержащих модуль. Рассмотрим некоторые из них.

1) Решение неравенства с помощью геометрического свойства модуля.

Напомню, что такое геометрическое свойство модуля: модуль числа x – это расстояние от начала координат до точки с координатой x.

В ходе решения неравенств этим способом может возникнуть 2 случая:

1. |x| ≤ b,

И неравенство с модулем очевидно сводится к системе двух неравенств. Тут знак может быть и строгим, в этом случае точки на картинке будут «выколотыми».

2. |x| ≥ b,
тогда картинка решения выглядит так:

И неравенство с модулем очевидно сводится к совокупности двух неравенств. Тут знак может быть и строгим, в этом случае точки на картинке будут «выколотыми».

Пример 1.

Решить неравенство |4 – |x||
3.

Решение.

Данное неравенство равносильно следующей совокупности:

U [-1;1] U

Пример 2.

Решить неравенство ||x+2| – 3|
2.

Решение.

Данное неравенство равносильно следующей системе.

{|x + 2| – 3 ≥ -2
{|x + 2| – 3 ≤ 2,
{|x + 2| ≥ 1
{|x + 2| ≤ 5.

Решим отдельно первое неравенство системы. Оно эквивалентно следующей совокупности:

U [-1; 3].

2) Решение неравенств, используя определение модуля.

Напомню для начала определение модуля.

|a| = a, если a
0 и |a| = -a, если a

Например, |34| = 34, |-21| = -(-21) = 21.

Пример 1.

Решить неравенство 3|x – 1|
x + 3.

Решение.

Используя определение модуля получим две системы:

{x – 1 ≥ 0
{3(x – 1) ≤ x + 3

{x – 1 {-3(x – 1) ≤ x + 3.

Решая первую вторую системы в отдельности, получим:

{x ≥ 1
{x ≤ 3,

{x {x ≥ 0.

Решением исходного неравенства будут все решения первой системы и все решения второй системы.

Ответ: x € .

3) Решение неравенств методом возведения в квадрат.

Пример 1.

Решить неравенство |x 2 – 1|

Решение.

Возведем обе части неравенства в квадрат. Замечу, что возводить обе части неравенства в квадрат можно только в том случае, когда они обе положительные. В данном случае у нас и слева и справа стоят модули, поэтому мы можем это сделать.

(|x 2 – 1|) 2

Теперь воспользуемся следующим свойством модуля: (|x|) 2 = x 2 .

(x 2 – 1) 2

(x 2 – 1) 2 – (x 2 – x + 1) 2

(x 2 – 1 – x 2 + x – 1)(x 2 – 1 + x 2 – x + 1)

(x – 2)(2x 2 – x)

x(x – 2)(2x – 1)

Решаем методом интервалов.

Ответ: x € (-∞; 0) U (1/2; 2)

4) Решение неравенств методом замены переменных.

Пример.

Решить неравенство (2x + 3) 2 – |2x + 3|
30.

Решение.

Заметим, что (2x + 3) 2 = (|2x + 3|) 2 . Тогда получим неравенство

(|2x + 3|) 2 – |2x + 3| ≤ 30.

Сделаем замену y = |2x + 3|.

Перепишем наше неравенство с учетом замены.

y 2 – y ≤ 30,

y 2 – y – 30 ≤ 0.

Разложим квадратный трехчлен, стоящий слева, на множители.

y1 = (1 + 11) / 2,

y2 = (1 – 11) / 2,

(y – 6)(y + 5) ≤ 0.

Решим методом интервалов и получим:

Вернемся к замене:

5 ≤ |2x + 3| ≤ 6.

Данное двойное неравенство равносильно системе неравенств:

{|2x + 3| ≤ 6
{|2x + 3| ≥ -5.

Решим каждое из неравенств в отдельности.

Первое равносильно системе

{2x + 3 ≤ 6
{2x + 3 ≥ -6.

Решим ее.

{x ≤ 1.5
{x ≥ -4.5.

Второе неравенство очевидно выполняется для всех x, так как модуль по определению число положительное. Так как решение системы – это все x, которые удовлетворяют одновременно и первому и второму неравенству системы, то решением исходной системы будет решение ее первого двойного неравенства (ведь второе верно для всех x).

Ответ: x € [-4,5; 1,5].

blog.сайт,
при полном или частичном копировании материала ссылка на первоисточник обязательна.

Презентация на тему: Неравенства с модулем.

Неравенства с модулями

11 класс презентация


0 равносильны двойному неравенству –a . Заметим, что если неравенство нестрогое |f(x) |≤ a , то при а=0 оно равносильно уравнению f(x)=0. Неравенства вида |f(x)| a при a ≤ 0 имеют решением всю область определения функции f(x) . При a 0 исходное неравенство равносильно двум неравенствам : f(x) a и f(x) , при этом решения обоих неравенств являются решениями исходного. Это справедливо и для нестрогих неравенств. «

Неравенства вида |f(x)| при a ≤ 0 решений не имеют, а при a 0 равносильны двойному неравенству –a . Заметим, что если неравенство нестрогое |f(x) |≤ a , то при а=0 оно равносильно уравнению f(x)=0.

Неравенства вида |f(x)| a при a ≤ 0 имеют решением всю область определения функции f(x) . При a 0 исходное неравенство равносильно двум неравенствам : f(x) a и f(x) , при этом решения обоих неравенств являются решениями исходного. Это справедливо и для нестрогих неравенств.


-3 Решение. Поскольку модуль всегда заведомо больше отрицательного числа, решением этого неравенства является область определения функции, стоящей под знаком модуля, т.е. любое рациональное число. Ответ : x (-∞;∞)   «

Решить неравенство :

|3 — 8x 2 | -3

Решение.

Поскольку модуль всегда заведомо больше отрицательного числа, решением этого неравенства является область определения функции, стоящей под знаком модуля, т.е. любое рациональное число.

Ответ : x (-∞;∞)

 

Решить неравенство: |x 2 — 2x| ≤ 0

Решение.

Решением этого неравенства будут корни уравнения x 2 -2x=0 ,т.е. x 1 =0 и x 2 =2 .

Ответ : x ϵ {0} ᴜ {2}

Решить неравенство : |7x 2 + 8| ≤ -3

Решение.

Неравенство решений не имеет, так как модуль всегда положителен.

Ответ : x ϵ

 

Решить неравенство : |x 2 — 7x| ≥ 12

Равносильные неравенства x 2 — 7x ≥ 12 и x 2 — 7x ≤ -12 сводятся к квадратным x 2 — 7x -12 ≥ 0 и x 2 — 7x + 12 ≤ 0 .

Решаем первое неравенство.

Корни уравнения 😡 1 = и x 2 = ; решением этого неравенства будут два полубесконечных интервала : (-∞;] и [ ;∞)

 

Решаем второе неравенство.

Корни уравнения: x 1 = 3 и x 2 = 4 ; решением этого неравенства будет интервал: x

 

Следовательно, решением исходного неравенства будут три интервала решений квадратных неравенств.

Ответ : x

 

Решить неравенство : | 3

 

Данное неравенство равносильно двойному :  -3 ≤ ≤ 3.

 

 

0

 

+ 3 ≥ 0

 

-3

 

3

 

0

 

— 3 ≤ 0

(6x – 8)(x – 3) ≥ 0

0

 

 

x ϵ ( -∞ ; ] ᴜ ( 3 ; ∞)

x ≠ 3

x ϵ ( — ∞ ; 3 )

 

0

x – 3

 

Ответ : x ϵ ( — ; )

Неравенства вида |f(x)|≤ g(x) сводятся к равносильной системе:

  • f(x) ≤ g(x) f(x) ≥ -g(x) ,

а неравенства вида |f(x)|≥ g(x) – к аналогичной равносильной системе:

f(x) ≥ g(x) f(x) ≤ -g(x) .


3 — x сводится к решению двух равносильных неравенств : x 1 = 3 x 2 = 6 x ϵ ( 3 ; 6 ) x ϵ ( -∞ ; 3) ᴜ ( 4 ; ∞ ) x 2 — 9x + 18 x 2 — 7x + 12 0 6 3 x 1 = 3 x 2 = 4 4 3 Ответ : x ϵ ( 4 ; 6 ) «

Решить неравенство : |x 2 — 8x + 15|

Решение .

Равносильная система неравенств

x 2 — 8x + 15 x 2 — 8x + 15 3 — x

сводится к решению двух равносильных неравенств :

x 1 = 3 x 2 = 6

x ϵ ( 3 ; 6 ) x ϵ ( -∞ ; 3) ᴜ ( 4 ; ∞ )

x 2 — 9x + 18 x 2 — 7x + 12 0

6

3

x 1 = 3 x 2 = 4

4

3

Ответ : x ϵ ( 4 ; 6 )


3x — 3 Решение . Равносильная система неравенств x 2 – 2x – 3 3x – 3 x 2 – 2x – 3 как и в предыдущем примере, сводится к системе квадратных неравенств : x 1 = 0 x 2 = 5 x ϵ ( -∞ ; 0 ) ᴜ ( 5 ; ∞ ) x ϵ ( -3 ; 2 ) x 2 –5x 0 x 2 + x – 6   0 5 x 1 = -3 x 2 = 2 2 -3 Ответ : x ϵ ( -3 ; 0 ) «

Решить неравенство : |x 2 – 2x — 3| 3x — 3

Решение .

Равносильная система неравенств

x 2 – 2x – 3 3x – 3 x 2 – 2x – 3

как и в предыдущем примере, сводится к системе квадратных неравенств :

x 1 = 0 x 2 = 5

x ϵ ( -∞ ; 0 ) ᴜ ( 5 ; ∞ ) x ϵ ( -3 ; 2 )

x 2 –5x 0 x 2 + x – 6

 

0

5

x 1 = -3 x 2 = 2

2

-3

Ответ : x ϵ ( -3 ; 0 )

Неравенства вида F(|f(x)|) v 0 заменой y=|f(x)| сводятся к равносильной системе :

F(y) v 0 y ≥ 0


0 – рациональное. Решать его нужно методом интервалов для рациональных и дробно-рациональных неравенств, предварительно разложив левую часть на множители. Корень у=1 кубического трехчлена угадывается сразу. Поэтому, деля y 3 + y — 2 на у – 1 или применяя метод неопределенных коэффициентов с использованием теоремы Безу, разложим выражение на два сомножителя – линейный и квадратный. Решить неравенство : |x — 4| 3 + |x — 4| 2 Решение . Заменой y=|x — 4| исходное неравенство сводится к равносильной системе : y 3 + y 2 y ≥ 0 y — 1 y 2 + y + 2 y 3 + y – 2 y 3 – y 2 — y 2 + y y 2 – y — 2у – 2 2у – 2 — 0 «

Первое неравенство y 3 + y — 2 0 рациональное. Решать его нужно методом интервалов для рациональных и дробно-рациональных неравенств, предварительно разложив левую часть на множители. Корень у=1 кубического трехчлена угадывается сразу. Поэтому, деля y 3 + y — 2 на у – 1 или применяя метод неопределенных коэффициентов с использованием теоремы Безу, разложим выражение на два сомножителя – линейный и квадратный.

Решить неравенство : |x — 4| 3 + |x — 4| 2

Решение .

Заменой y=|x — 4| исходное неравенство сводится к равносильной системе :

y 3 + y 2 y ≥ 0

y — 1 y 2 + y + 2

y 3 + y – 2 y 3 – y 2

y 2 + y y 2 – y

2у – 2 2у – 2

0


0 y≥0 , откуда у1 Переходя к переменной х, получаем простейшее неравенство |х — 4| 1 , которое разбивается на два равносильных : х – 4 1 , или х 5 , и х – 4 , или х . Таким образом, x ϵ ( -∞ ; 3) ᴜ ( 5; ∞ ) . Ответ : x ϵ ( -∞ ; 3) ᴜ ( 5 ; ∞ ) «

Таким образом,

y 3 + y – 2 = (у – 1)(y 2 + y + 2)

Итак, равносильная система приняла вид :

(у – 1)(y 2 + y + 2)0 y≥0 ,

откуда у1

Переходя к переменной х, получаем простейшее неравенство |х — 4| 1 , которое разбивается на два равносильных : х – 4 1 , или х 5 , и х – 4 , или х . Таким образом, x ϵ ( -∞ ; 3) ᴜ ( 5; ∞ ) .

Ответ : x ϵ ( -∞ ; 3) ᴜ ( 5 ; ∞ )

  • Неравенства вида F ( ϕ (x) ; | f(x) | ) v 0 сводятся к двум равносильным системам :

f(x) 0 F( ϕ (x) ; f(x)) v 0

f(x) 0 F( ϕ (x) ; -f(x)) v 0

Решить неравенство : x 2 + 2|x + 3| — 10 ≤ 0

Решение .

Равносильные системы :

 

x ≥ -3 x 1 = -1 + x 2 = -1 —

x ≥ -3 x 2 + 2x – 4 ≤ 0

x + 3 ≥ 0 x 2 + 2(x + 3) – 10 ≤ 0

x ϵ [-3 ; -1 +

 

и

x x 1 = 1 + x 2 = 1 —

 

x + 3 x 2 — 2(x + 3) – 10 ≤ 0

x x 2 — 2x – 16 ≤ 0

 

x ϵ [1 — ; -1 + ]

Ответ : x ϵ [ 1 — ; -1 + ]

 


|g(x)| , как и соответствующие уравнения, сводятся к равносильному [ f(x) ] 2 [ g(x) ] 2 . «

Неравенства вида |f(x)||g(x)| , как и соответствующие уравнения, сводятся к равносильному [ f(x) ] 2 [ g(x) ] 2 .

Решить неравенство : |2x — 5| — |4x + 7|≥ 0

Решение .

Приводим исходное неравенство к виду

|2x — 5| ≥ |4x + 7|

и возводим в квадрат :

(2x – 5) 2 ≥ (4x + 7) 2

 

Решением неравенства является интервал [ -6 ; — ] , он же – решением исходного неравенства с модулями.

Раскрывая скобки и приводя подобные члены, получаем квадратное неравенство :

3x 2 + 19x + 6 ≤ 0

 

x 1 = — 6 x 2 = —

3x 2 + 19x + 6 = 0

Ответ : x ϵ [ -6 ; — ]

 

Не следует путать этот метод с методом интервалов, применяемым для решения рациональных и дробно-рациональных неравенств.

Внимание!

Неравенства вида |f 1 (x)|±|f 2 (x)|±…±|f n (x)| v a решаются тем же самым методом интервалов, что и уравнения с модулем. Разница лишь в том, что в данном случае в каждом интервале решается не уравнение, а неравенство и из решений неравенства выбираются те, которые принадлежат данному интервалу. В остальном метод интервалов остается тем же, что и при решении уравнений с модулем.


0 x x + 1 0 x + 2 0 x0 x + 1 0 x + 2 0 Раскроем модули на интервале I (x≤-2) : -x + 2( + 1) – 3(x + 2) ≥ 4 После преобразования получаем -2х – 4 ≥ 4 , откуда х ≤ -4 «

Решить неравенство : |x|- 2|x + 1|+ 3|x + 2|≥ 4

Решение .

Найдем сначала все x i , разбивающие числовую ось на интервалы и получающиеся как решения уравнений f j (x i )=0 : х=0; х + 1=0 ; х + 2=0 . Таким образом, границами интервалов являются числа x 1 =0, x 2 =-1 и x 3 =-2. Отметим эти значения на числовой оси для каждого из полученных интервалов определим знаки выражений, стоящих под знаком модуля :

III

IV

II

I

-2

-1

0

x x + 1 x + 2

x x + 1 x + 2 0

x x + 1 0 x + 2 0

x0 x + 1 0 x + 2 0

Раскроем модули на интервале I (x≤-2) :

-x + 2( + 1) – 3(x + 2) ≥ 4

После преобразования получаем

-2х – 4 ≥ 4 ,

откуда х ≤ -4

Этот интервал входит в интервал I и является решением исходного неравенства.

II интервал ( -2≤ х ≤-1 ) :

-х+ 2(х + 1) + 3(х + 2) ≥ 4

4х + 8 ≥ 4 ,

откуда х≥-1

Решением в этом интервале является точка х=-1

III интервал ( -1≤ х ≤0 ) :

-х – 2(х + 1) + 3(х + 2) ≥ 4

4 ≥ 4

В результате мы получили истинное неравенство ( заметим, что, если бы неравенство было строгим, оно становилось бы ложным). Следовательно, весь интервал III является решением исходного неравенства.

IV интервал ( х≥0 )

х – 2(х + 1) + 3( + 2) ≥ 4

2х + 4 ≥ 4 ,

откуда х≥0

Таким образом, весь интервал IV является решением исходного неравенства. Заметим, что решения, полученные в интервалах II , III и IV , «сливаются» по граничным точкам x 1 =-1 и x 2 =0 в единый интервал [-1 ; ∞).

 

Ответ : ( -∞ ; -4 ] ᴜ [ -1 ; ∞ )

Совокупности неравенств, неравенства с модулем

п.1. Понятие совокупности неравенств с одной переменной и его решения

Несколько неравенств с одной переменной образуют совокупность, если нужно найти такое множество значений переменной, которое будет решением хотя бы одного из неравенств.
Решением совокупности неравенств с одной переменной является такое множество значений этой переменной, которое превращает хотя бы одно из неравенств в верное числовое неравенство.2 \geq 0} & \\ \mathrm{x-4\lt 1} & \end{array}\right. \Leftrightarrow \left[ \begin{array}{ l l } \mathrm{x\leq-2\cup x \geq 2} & \\ \mathrm{x\lt 5} & \end{array}\right. \Leftrightarrow \mathrm{x \in \mathbb{R}} $ — любое действительное число.

п.2. Алгоритм решения совокупности неравенств с одной переменной

Шаг 1. Найти множество решений для каждого из неравенств системы. Если какое-либо частное решение является пустым множеством, отбросить его, но продолжить решение.

Шаг 2. Начертить друг под другом числовые прямые, число которых равно числу полученных непустых частных решений. Начала отсчёта числовых прямых должны находиться на общем перпендикуляре, единичный отрезок должен совпадать.

Шаг 3. На числовых прямых изобразить полученные частные решения, на отдельной прямой найти их объединение – это и будет общим решением системы.

Шаг 4. Работа завершена.

Например: $ \left[ \begin{array}{ l l } \mathrm{x^2-1 \lt 0} & \\ \mathrm{x+5\geq8} & \end{array}\right. \Leftrightarrow \left[ \begin{array}{ l l } \mathrm{-1\lt x\lt 1} & \\ \mathrm{x\geq 3} & \end{array}\right. \Leftrightarrow \mathrm{-1\lt x \lt 1\cup x\geq 3} $
или $\mathrm{x\in (-1;1)\cup\left[3;+\infty\right)}$

Подробней о сравнении систем и совокупностей неравенств, их соответствии логическим операциям, см. §39 справочника для 8 класса.

п.3. Решение неравенств с модулем

Пусть f(x) — некоторая функция от x, a – некоторое действительное число. Составим таблицу возможных неравенств с модулем и их решений:

Решений нет, $\mathrm{x\in\varnothing}$

Выполняется всегда, $\mathrm{x\in\mathbb{R}}$

Решений нет, $\mathrm{x\in\varnothing}$

Решаем неравенство: $\mathrm{f(x)\ne 0}$

Решаем двойное неравенство: $\mathrm{-a\lt f(x)\lt a}$
т.е. систему: $ \left\{ \begin{array}{ l l } \mathrm{f(x)\gt -a} & \\ \mathrm{f(x)\lt a} & \end{array}\right. $

Решаем совокупность: $ \left[ \begin{array}{ l l } \mathrm{f(x)\lt -a} & \\ \mathrm{f(x)\gt a} & \end{array}\right.2+4x\gt 0} & \end{array}\right. \end{gather*}

y = x2 + 2x + 4 > 0 — парабола с a > 0 и D < 0, т.е. все её точки лежат над осью OX. Значит, решение первого неравенства – пустое множество, \(x\in\varnothing\).
Решаем второе неравенство: $$ \mathrm{x(x+4) \gt 0\Rightarrow x\lt -4\cup x\gt 0} $$ Ответ: \(\mathrm{x\in(-\infty;-4)\cup(0;+\infty)}\)

п.4. Примеры

Пример 1. Решите неравенства:
а) |x + 2| > 5
Решаем совокупность: \begin{gather*} \left[ \begin{array}{ l l } \mathrm{x+2\gt-5} & \\ \mathrm{x+2\gt 5} & \end{array}\right.\Rightarrow \left[ \begin{array}{ l l } \mathrm{x\lt -7} & \\ \mathrm{x\gt 3} & \end{array}\right.\Rightarrow x\lt -7\cup x\gt 3 \end{gather*} Ответ: \(\mathrm{x\in(-\infty;-7)\cup(3;+\infty)}\)

б) |22 — 7x| ≤ 6
Решаем систему: \begin{gather*} \left\{ \begin{array}{ l l } \mathrm{22-7x\geq -6} & \\ \mathrm{22-7x\leq 6} & \end{array}\right.\Rightarrow \left\{ \begin{array}{ l l } \mathrm{-7x\geq -28} & \\ \mathrm{-7x\leq -16} & \end{array}\right.2+8x+12\gt 0} & \end{array}\right. \Rightarrow \left[ \begin{array}{ l l } \mathrm{x=3}& \\ \mathrm{(x+2)(x+6)\gt 0} & \end{array}\right. \Rightarrow \left[ \begin{array}{ l l } \mathrm{x=3}& \\ \mathrm{-6\lt x\lt -2} & \end{array}\right.\Rightarrow \\ \Rightarrow \mathrm{-6\lt x\lt -2\cup x=3} \end{gather*}

Ответ: \(\mathrm{x\in(-6;-2)\cup{3}}\).

e) \(\mathrm{\frac{4}{|x+2|}\lt\frac{3x+23}{5}}\)
Решаем систему:
\begin{gather*} \left\{ \begin{array}{ l l } \mathrm{\frac{4}{x+2}\gt -\frac{3x+23}{5}} & \\ \mathrm{\frac{4}{x+2}\lt\frac{3x+23}{5}} & \end{array}\right. \Rightarrow \left\{ \begin{array}{ l l } \mathrm{\frac{4}{x+2}+\frac{3x+23}{5}\gt 0}& \\ \mathrm{\frac{4}{x+2}-\frac{3x+23}{5}\lt 0} & \end{array}\right. \Rightarrow \left\{ \begin{array}{ l l } \mathrm{\frac{20+(3x+23)(x+2)}{5(x+2)}}& \\ \mathrm{\frac{20-(3x+23)(x+2)}{5(x+2)}} & \end{array}\right.\Rightarrow \\ \Rightarrow \left\{ \begin{array}{ l l } \mathrm{\frac{3x^2+29x+66}{5(x+2)}\gt 0}& \\ \mathrm{\frac{-3x^2-29x-26}{5(x+2)}\lt 0} & \end{array}\right.2,\ \ x_{3,4}=\frac{-29\pm 23}{6}}=\left[ \begin{array}{ l l } \mathrm{-8\frac23}& \\ \mathrm{-1} & \end{array}\right. \end{gather*}

Получаем:
\begin{gather*} \left\{ \begin{array}{ l l } \mathrm{\frac{(c+6)\left(x+3\frac23\right)}{x+2}\gt 0} & \\ \mathrm{\frac{\left(x+8\frac23\right)(x+1)}{x+2}\gt 0} & \end{array}\right. \end{gather*}

Ответ: \(\mathrm{x\in\left(-6;-3\frac23\right)\cup(-1;\infty)}\).

4. Неравенства, связанные с абсолютными значениями

Для неравенств, включающих абсолютные значения, т.е. | x |, мы используем следующие отношения, для некоторого числа n :

Если `| f (x) | > n`, то это означает:

`f (x) <-n` или` f (x)> n`

Если `| f (x) |

`-n < f (x)

Пример 1

Решите неравенство | x — 3 | <2.

Ответ

Применение отношений
обсуждалось выше:

`- 2

Добавляя `3` ко всем сторонам, получаем:

`-2 + 3

`1

Вот график нашего решения:

Убедитесь, что это правильный ответ, проверив. Попробуйте `x = 0` (не удастся, потому что это выходит за пределы диапазона нашего ответа),` x = 3` (должно работать) и `x = 10` (должно потерпеть неудачу).Каждый раз, когда вы так проверяете, становится понятнее, почему мы решаем именно так.

Пример 2

Решите неравенство | 2 x — 1 | > 5.

Ответ

Применяя отношения, обсужденные ранее:

`2x — 1 <-5 \ или \ 2x - 1> 5`

Решая оба неравенства, получаем:

`2x <-5 + 1` или `2x> 5 + 1`
`2x <-4` или `2x> 6`
`x <-2` или `x> 3`

Вот график нашего решения:

Пример 3

Решите неравенство `2 | (2x) / 3 + 1 |> = 4`

Ответ

`2 | (2x) / 3 + 1 |> = 4`

Это дает

`| (2x) / 3 + 1 |> = 2`

Так что либо…

`(2x) / 3 + 1 <= - 2`

ИЛИ

`(2x) / 3 + 1> = 2`

Умножить обе стороны на 3.

`2x + 3 ≤ -6
`

`2x ≤ -9`

`x <= - 9/2`

ИЛИ

`2x + 3 ≥ 6
`

`2x ≥ 3`

`x> = 3/2`

Решение: `x <= - 9/2 = -4,5, \" или "\ x> = 3/2 = 1,5`

Вот график нашего решения:

Пример 4

Решите неравенство `| 3 — 2x | <3`

Ответ

`| 3 — 2x | <3`

`-3 <3 - 2x <3`

Вычтем `3` со всех сторон:

`-6 <-2x <0`

Разделить все стороны на `-2`

`3> x> 0`

(обратите внимание на изменение смысла из-за деления на отрицательное
номер)

Лучше записать это: `0

Вот график решения:

Пример 5

Техник измеряет электрический ток, который составляет `0,036 \» A «`
с возможной погрешностью `± 0,002 \» A «`. Напишите это
ток, и , как неравенство с абсолютным
значения.

Ответ

Возможная ошибка `± 0,002 \» A «означает, что разница между фактическим током и значением
`0,036 \» A «` не может быть больше `0,002 \» A «`.

Значения и , которые у нас есть, могут быть выражены как:

`0,034 ≤ i ≤ 0,038`

Мы можем просто записать это как:

`| i — 0,036 | ≤ 0,002`

Вот график решения:

Упражнения

1. Решить `| 5 — x | ≤ 2`

Ответ

`-2 ≤ 5 — x ≤ 2`

`-7 ≤ — x ≤ -3`

`7 ≥ x ≥ 3`

Лучше записать это как `3 ≤ x ≤ 7`.

Вот график решения:

2. Решить `| (2x-9) / 4 | <1`

Ответ

`-1 <(2x-9) / 4 <1`

«-4 <2x-9 <4"

`5 <2x <13`

`2,5

3. Решить `| (4x) / 3-5 |> = 7`

Ответ

`| (4x) / 3-5 |> = 7`

`(4x) / 3-5 <= - 7` ИЛИ `(4x) / 3-5> = 7`

`4x-15 <= - 21`

`4x <= - 6`

`x <= - 1.2 + 3x-4 <0`

`(x + 4) (x-1) <0`

Критические значения:

`x = -4` и` x = 1`

Это дает:

`-4

Проще изобразить их вместе на одной оси, чтобы определить окончательный результат:

Эти 3 региона пересекаются в следующих 2 местах:

Это дает нам окончательное решение: −4 < x <−2 и −1 < x <1.

Не забудьте проверить номеров внутри и за пределами данной области решения, чтобы убедиться, что оно работает.

Абсолютное значение и неравенства — GMAT Math Study Guide

Определения

Неравенство сравнивает два значения.

  • Неравенство — сравнение двух значений или выражений.
    Например, 15x <45 - это неравенство, а x = 3 - это уравнение.
  • Equation — Утверждение, объявляющее равенство двух выражений.
    Например, 25x = 50 — это уравнение, а 25x> 50 — неравенство.

Работа с неравенствами: абсолютное значение

Так же, как традиционные уравнения с членом абсолютного значения в них обычно имеют два решения, так и неравенства с членом абсолютного значения в них обычно имеют два решения. Чтобы решить неравенство с термином абсолютного значения, выделите абсолютное значение, установите выражение в скобках абсолютного значения, равное положительному и отрицательному значению, и найдите неизвестную переменную.

1.) Выделите член абсолютного значения
10 + | 2x + 15 |
10-10 + | 2x + 15 |
| 2x + 15 |

2.) Установите выражение в члене абсолютного значения, равное как положительному, так и отрицательному значению

Положительное: 2x + 15 <45
2x + 15-15 <45-15
2x <30
x <15

Отрицательное : — (2x + 15) <45
-2x — 15 <45
-2x — 15 + 15 <45 + 15
-2x <60
x> -30 [Знак неравенства меняется, потому что мы делим на отрицательное число]

В качестве проверки, если x = 30 (вне набора решений), 10 + | 2x + 15 | = 85, что не укладывается в неравенство (т.е., 85 не меньше 55). Однако, если x = -10 (внутри набора решений), 10 + | 2x + 15 | = 15, что соответствует неравенству (т.е. 15 меньше 55).

Множественные неравенства

Процесс решения множественных неравенств с абсолютным значением такой же, как и процесс решения множественных неравенств без абсолютного значения. При решении нескольких одновременных неравенств с абсолютным значением наиболее важной частью является решение каждого неравенства отдельно, а затем их объединение.

Если | x + 7 | > 16, каков диапазон возможных значений x?

1.) Решите каждое неравенство отдельно.
| x + 7 | <14
Положительный: x + 7 <14
x + 7-7 <14-7
x <7
Отрицательный: -1 (x + 7) <14
-x — 7 <14
-x — 7 + 7 <14 + 7
-x <21
x> -21

| 4x — 4 | > 16
Положительный: 4x — 4> 16
4x — 4 + 4> 16 + 4
4x> 20
x> 5
Отрицательный: -1 (4x — 4)> 16
-4x + 4> 16
-4x + 4-4> 16-4
-4x> 12
x <-3

2.) Объедините каждое неравенство и найдите перекрытие (т. Е. Области, в которых выполняется каждое неравенство — эта область является решением).
x <7
x> -21
x> 5
x <-3

Есть две области, в которых выполняется каждое неравенство: (1) -21

Для многих студентов вышеуказанный набор неравенств лучше всего можно понять графически. Решением множества неравенств является перекрывающаяся графическая область. Красные линии представляют решения | 4x — 4 | > 16, а синие линии представляют решение | x + 7 | <14 неравенство.

2.7 Линейные неравенства и абсолютные неравенства — College Algebra

Рисунок 1

Попасть в список почета в большинстве ведущих университетов непросто. Предположим, студенты должны были пройти курс не менее 12 кредитных часов и поддерживать средний балл 3,5 или выше. Как можно математически выразить эти требования к списку почета? В этом разделе мы рассмотрим различные способы выражения различных наборов чисел, неравенств и неравенств по абсолютным значениям.

Использование записи интервалов

Указать решение неравенства, такого как x≥4x≥4, можно несколькими способами.

Мы можем использовать числовую линию, как показано на рисунке 2 . Синий луч начинается с x = 4x = 4 и, как показано стрелкой, продолжается до бесконечности, что показывает, что набор решений включает все действительные числа, большие или равные 4.

Рисунок 2

Мы можем использовать нотацию построителя множеств: {x | x≥4}, {x | x≥4}, что переводится как «все действительные числа x , такие, что x больше или равно 4.Обратите внимание, что фигурные скобки используются для обозначения набора.

Третий метод — это интервальная запись, в которой наборы решений указываются круглыми или квадратными скобками. Решения x≥4x≥4 представлены как [4, ∞). [4, ∞). Это, пожалуй, самый полезный метод, поскольку он применим к концепциям, изучаемым позже в этом курсе, и к другим курсам математики более высокого уровня.

Основная концепция, которую следует запомнить, заключается в том, что круглые скобки представляют решения, которые больше или меньше числа, а скобки представляют решения, которые больше или равны или меньше или равны числу.Используйте круглые скобки для обозначения бесконечности или отрицательной бесконечности, поскольку положительная и отрицательная бесконечность не являются числами в обычном смысле слова и, следовательно, не могут быть «равны». Несколько примеров интервала или набора чисел, в которые попадает решение: [−2,6), [- 2,6) или все числа от −2−2 до 6,6, включая −2, −2, но не включая 6; 6; (−1,0), (- 1,0), все действительные числа между, но не включая −1−1 и 0; 0; и (−∞, 1], (- ∞, 1], все действительные числа, меньшие, чем 1.1, включая 1.1. В таблице 1 показаны возможные варианты.

Указанный набор Обозначение конструктора множеств Интервальное обозначение
Все действительные числа от a до b , за исключением a или b {x | a (а, б) (а, б)
Все действительные числа больше a , но не включая a {x | x> a} {x | x> a} (а, ∞) (а, ∞)
Все действительные числа меньше b , но не включая b {x | x (−∞, б) (- ∞, б)
Все действительные числа больше a , включая a {x | x≥a} {x | x≥a} [а, ∞) [а, ∞)
Все действительные числа меньше b , в том числе b {x | x≤b} {x | x≤b} (−∞, b] (- ∞, b]
Все вещественные числа от a до b , включая a {x | a≤x [а, б) [а, б)
Все вещественные числа от a до b , включая b {x | a (a, b] (a, b]
Все вещественные числа от a до b , включая a и b {x | a≤x≤b} {x | a≤x≤b} [a, b] [a, b]
Все действительные числа меньше a или больше b {x | x b} {x | x b} (−∞, a) ∪ (b, ∞) (- ∞, a) ∪ (b, ∞)
Все вещественные числа {x | x — все действительные числа} {x | x — все действительные числа} (−∞, ∞) (- ∞, ∞)

Таблица 1

Пример 1

Использование интервальной записи для выражения всех действительных чисел, больше или равных

a

Используйте обозначение интервала, чтобы указать все действительные числа, большие или равные -2.−2.

Решение

Используйте квадратную скобку слева от −2−2 и круглые скобки после бесконечности: [−2, ∞). [- 2, ∞). Скобка указывает, что −2−2 включен в набор со всеми действительными числами от −2−2 до бесконечности.

Попробуй # 1

Используйте обозначение интервала для обозначения всех действительных чисел от −3−3 до 5,5 включительно.

Пример 2

Использование интервальной записи для выражения всех действительных чисел, меньших или равных

a или больше или равных b

Запишите интервал, выражающий все действительные числа, меньшие или равные -1-1 или больше или равные 1.1.

Решение

Для этого примера мы должны написать два интервала. Первый интервал должен указывать все действительные числа, меньшие или равные 1. Итак, этот интервал начинается с −∞ − ∞ и заканчивается на −1, −1, что записывается как (−∞, −1]. (- ∞, −1].

Во втором интервале должны отображаться все действительные числа, большие или равные 1,1, что записывается как [1, ∞). [1, ∞). Однако мы хотим объединить эти два набора. Мы достигаем этого, вставляя символ объединения, ∪, ∪, между двумя интервалами.

(−∞, −1] ∪ [1, ∞) (- ∞, −1] ∪ [1, ∞)

Попробуй # 2

Выразите все действительные числа меньше -2-2 или больше или равные 3 в виде интервалов.

Использование свойств неравенств

Когда мы работаем с неравенствами, мы обычно можем относиться к ним аналогично, но не точно так, как мы относимся к равенствам. Мы можем использовать свойство сложения и свойство умножения, чтобы помочь нам решить их. Единственное исключение — это когда мы умножаем или делим на отрицательное число; при этом символ неравенства меняется на противоположное.

Свойства неравенств

AdditionProperty Если a 0, то ac bc.AdditionProperty Если a 0, то ac bc.

Эти свойства также применимы к a≤b, a≤b, a> b, a> b и a≥b.a≥b.

Пример 3

Демонстрация свойства сложения

Проиллюстрируйте свойство сложения для неравенств, решив каждое из следующих решений:

  1. ⓐ x − 15 <4x − 15 <4

  2. 6≥x − 16≥x − 1

  3. х + 7> 9 х + 7> 9
Решение

Свойство сложения для неравенств гласит, что если неравенство существует, добавление или вычитание одного и того же числа с обеих сторон не меняет неравенства.


  1. x − 15 <4x − 15 + 15 <4 + 15 Прибавить 15 к обеим сторонам. X <19x − 15 <4x − 15 + 15 <4 + 15 Прибавить 15 к обеим сторонам. X <19

  2. 6≥x − 16 + 1≥x − 1 + 1 Прибавить 1 к обеим сторонам 7≥x6≥x − 16 + 1≥x − 1 + 1 Прибавить 1 к обеим сторонам 7≥x

  3. x + 7> 9x + 7−7> 9−7 Вычтите 7 с обеих сторон. X> 2x + 7> 9x + 7−7> 9−7 Вычтите 7 с обеих сторон. X> 2

Попробуй # 3

Решить: 3x − 2 <1,3x − 2 <1.

Пример 4

Демонстрация свойства умножения

Проиллюстрируйте свойство умножения неравенств, решив каждую из следующих задач:

  1. ⓐ 3x <63x <6
  2. ⓑ −2x − 1≥5−2x − 1≥5
  3. ⓒ 5-х> 105-х> 10
Решение

  1. 3x <613 (3x) <(6) 13x <23x <613 (3x) <(6) 13x <2

  2. −2x − 1≥5−2x≥6 (−12) (- 2x) ≥ (6) (- 12) Умножить на −12.x≤ − 3 Отмените неравенство. −2x − 1≥5−2x≥6 (−12) (- 2x) ≥ (6) (- 12) Умножьте на −12.x≤ − 3 Отмените неравенство.

  3. 5 − x> 10 − x> 5 (−1) (- x)> (5) (- 1) Умножить на −1.x <−5 Отменить неравенство. 5 − x> 10 − x> 5 ( −1) (- x)> (5) (- 1) Умножить на −1.x <−5 Отменить неравенство.

Попробуй # 4

Решите: 4x + 7≥2x − 3.4x + 7≥2x − 3.

Алгебраическое решение неравенств с одной переменной

Как показали примеры, мы можем выполнять те же операции с обеими сторонами неравенства, как и с уравнениями; совмещаем похожие сроки и выполняем операции.Чтобы решить, мы изолируем переменную.

Пример 5

Алгебраическое решение неравенства

Решите неравенство: 13−7x≥10x − 4.13−7x≥10x − 4.

Решение

Решение этого неравенства аналогично решению уравнения до последнего шага.

13−7x≥10x − 413−17x≥ − 4 Переместите члены переменной в одну сторону неравенства. − 17x≥ − 17 Выделите член переменной. X≤1 Разделение обеих частей на −17 отменяет неравенство. ≥ − 4Переместите переменные члены в одну сторону неравенства.−17x≥ − 17 Выделите член переменной. X≤1 Разделение обеих частей на −17 отменяет неравенство.

Множество решений задается интервалом (−∞, 1], (- ∞, 1] или всеми действительными числами, меньшими 1 включительно.

Попробуй # 5

Решите неравенство и запишите ответ в интервале: −x + 4 <12x + 1. − x + 4 <12x + 1.

Пример 6

Решение неравенства с дробями

Решите следующее неравенство и запишите ответ в интервале: −34x≥ − 58 + 23x.−34x≥ − 58 + 23x.

Решение

Мы начинаем решать так же, как и при решении уравнения.

−34x≥ − 58 + 23x − 34x − 23x≥ − 58 Положите переменные члены в одну сторону. −912x − 812x≥ − 58 Запишите дроби с общим знаменателем. − 1712x≥ − 58x≤ − 58 (−1217) Умножение на отрицательное число меняет местами неравенство. x≤1534−34x≥ − 58 + 23x − 34x − 23x≥ − 58 Положите переменные члены в одну сторону. −912x − 812x≥ − 58 Запишите дроби с общим знаменателем. −1712x≥ − 58x≤ − 58 (−1217) Умножение на отрицательное число отменяет неравенство.x≤1534

Множество решений — это интервал (−∞, 1534]. (- ∞, 1534].

Попробуй # 6

Решите неравенство и запишите ответ в интервале: −56x≤34 + 83x. − 56x≤34 + 83x.

Понимание сложных неравенств

Сложное неравенство включает в себя два неравенства в одном утверждении. Утверждение, такое как 4

Пример 7

Решение сложного неравенства

Решите составное неравенство: 3≤2x + 2 <6.3≤2x + 2 <6.

Решение

Первый способ — написать два отдельных неравенства: 3≤2x + 23≤2x + 2 и 2x + 2 <6.2x + 2 <6. Решаем их самостоятельно.

3≤2x + 2and2x + 2 <61≤2x2x <412≤xx <23≤2x + 2and2x + 2 <61≤2x2x <412≤xx <2

Затем мы можем переписать решение как составное неравенство, таким же образом проблема началась.

В интервальной записи решение записывается как [12,2). [12,2).

Второй метод — оставить составное неравенство неповрежденным и выполнить процедуры решения для трех частей одновременно.

3≤2x + 2 <61≤2x <4 Выделите член переменной и вычтите 2 из всех трех частей. 12≤x <2D Разделите все три части на 2,3≤2x + 2 <61≤2x <4 Выделите член переменной и вычтите 2 из всех трех частей. 12≤x <2D Разделим все три части на 2.

Получим такое же решение: [12,2).[12,2).

Попробуйте # 7

Решите составное неравенство: 4 <2x − 8≤10,4 <2x − 8≤10.

Пример 8

Решение сложного неравенства с переменной во всех трех частях

Решите составное неравенство с переменными во всех трех частях: 3 + x> 7x − 2> 5x − 10,3 + x> 7x − 2> 5x − 10.

Решение

Попробуем первый способ. Запишем два неравенства :

3 + x> 7x − 2 и 7x − 2> 5x − 103> 6x − 22x − 2> −105> 6x2x> −856> xx> −4x <56−4 7x− 2and7x − 2> 5x − 103> 6x − 22x − 2> −105> 6x2x> −856> xx> −4x <56−4 Набор решений равен −4 .

Рисунок 3

Попробуй # 8

Решите составное неравенство: 3y <4−5y <5 + 3y.3y <4−5y <5 + 3y.

Решение абсолютных неравенств

Как мы знаем, абсолютное значение величины — это положительное число или ноль.От начала координат точка, расположенная в (−x, 0) (- x, 0), имеет абсолютное значение x, x, так как она находится на расстоянии x единиц. Считайте абсолютное значение расстоянием от одной точки до другой точки. Независимо от направления, положительного или отрицательного, расстояние между двумя точками представляется как положительное число или ноль.

Неравенство по абсолютной величине — это уравнение вида

| A | B или | A | ≥B, | A | B или | A | ≥B,

Где A , а иногда B , представляет алгебраическое выражение, зависящее от переменной x. Решение неравенства означает нахождение набора всех xx значений, удовлетворяющих задаче. Обычно этот набор представляет собой интервал или объединение двух интервалов и включает диапазон значений.

Существует два основных подхода к решению абсолютных неравенств: графический и алгебраический. Преимущество графического подхода в том, что мы можем прочитать решение, интерпретируя графики двух уравнений. Преимущество алгебраического подхода состоит в том, что решения являются точными, поскольку точные решения иногда трудно прочитать с графика.

Предположим, мы хотим знать все возможные доходы от инвестиций, если бы мы могли заработать некоторую сумму денег в пределах от 200 до 600 долларов. Мы можем решить алгебраически для набора из x- значений, так что расстояние между xx и 600 меньше или равно 200. Мы представляем расстояние между xx и 600 как | x − 600 |, | x − 600 |, и, следовательно, | x − 600 | ≤200 | x − 600 | ≤200 или

−200≤x − 600≤200−200 + 600≤x − 600 + 600≤200 + 600400≤x≤800−200≤x− 600≤200−200 + 600≤x − 600 + 600≤200 + 600400≤x≤800

Это означает, что наша прибыль составит от 400 до 800 долларов США.

Для решения неравенств по абсолютным значениям, как и для уравнений с абсолютными значениями, мы записываем два неравенства и затем решаем их независимо.

Абсолютные неравенства

Для алгебраического выражения X, и k> 0, k> 0 неравенство по модулю является неравенством вида

| X | k эквивалентно X <−kor X> k | X | k эквивалентно X <- kor X> k

Эти утверждения также применимы к | X | ≤k | X | ≤k и | X | ≥k.| X | ≥k.

Пример 9

Определение числа на заданном расстоянии

Опишите все значения xx на расстоянии 4 от числа 5.

Решение

Мы хотим, чтобы расстояние между xx и 5 было меньше или равно 4. Мы можем нарисовать числовую линию, например, как на рис. 4 , , чтобы представить условие, которое должно быть выполнено.

Рисунок 4

Расстояние от xx до 5 может быть представлено с помощью символа абсолютного значения | x − 5 |.| х − 5 |. Запишите значения xx, которые удовлетворяют условию, как неравенство по абсолютной величине.

Нам нужно написать два неравенства, так как всегда есть два решения уравнения абсолютного значения.

x − 5≤4andx − 5≥ − 4x≤9x≥1x − 5≤4andx − 5≥ − 4x≤9x≥1

Если набор решений x≤9x≤9 и x≥1, x≥1, то решение set — интервал, включающий все действительные числа от 1 до 9 включительно.

Итак, | x − 5 | ≤4 | x − 5 | ≤4 эквивалентно [1,9] [1,9] в обозначении интервалов.

Попробуй # 9

Опишите все значения x- на расстоянии 3 от числа 2.

Пример 10

Разрешение абсолютного неравенства значений

Решить | x − 1 | ≤3 | x − 1 | ≤3.

Решение

| x − 1 | ≤3−3≤x − 1≤3−2≤x≤4 [−2,4] | x − 1 | ≤3−3≤x − 1≤3−2≤x≤4 [- 2,4]

Пример 11

Использование графического подхода для решения абсолютных неравенств

Учитывая уравнение y = −12 | 4x − 5 | + 3, y = −12 | 4x − 5 | +3, определите значения x , для которых значения y отрицательны.

Решение

Мы пытаемся определить, где y <0, y <0, то есть когда −12 | 4x − 5 | +3 <0.−12 | 4х − 5 | +3 <0. Начнем с выделения абсолютного значения.

−12 | 4x − 5 | <−3 Умножьте обе части на –2 и отмените неравенство. | 4x − 5 |> 6−12 | 4x − 5 | <−3 Умножьте обе части на –2 и измените неравенство в обратном порядке. | 4x − 5 |> 6

Затем мы решаем равенство | 4x − 5 | = 6. | 4x − 5 | = 6.

4x − 5 = 64x − 5 = −64x = 11or4x = −1x = 114x = −144x − 5 = 64x − 5 = −64x = 11or4x = −1x = 114x = −14

Теперь мы можем изучить график, чтобы увидеть где значения y- отрицательны. Мы наблюдаем, где ветви находятся ниже оси x-.Обратите внимание, что не важно, как именно выглядит график, если мы знаем, что он пересекает горизонтальную ось в точках x = −14x = −14 и x = 114, x = 114, и что график открывается вниз. См. Рисунок 5 .

Рисунок 5

Попробуй # 10

Решить −2 | k − 4 | ≤ − 6. − 2 | k − 4 | ≤ − 6.

2.7 Раздел упражнений

Устный

1.

При решении неравенства объясните, что произошло с шага 1 по шаг 2:

Шаг 1-2x> 6 Шаг 2x <−3 Шаг 1-2x> 6 Шаг 2x <−3

2.

Решая неравенство, получаем:

х + 2 <х + 32 <3х + 2 <х + 32 <3

Объясните, что представляет собой наш набор решений.

3.

Как мы представляем все действительные числа при записи нашего решения в интервальной нотации?

4.

Решая неравенство, получаем:

х + 2> х + 32> 3х + 2> х + 32> 3

Объясните, что представляет собой наш набор решений.

5.

Опишите, как построить график y = | x − 3 | y = | x − 3 |

Алгебраические

Для следующих упражнений решите неравенство.Напишите окончательный ответ в виде интервалов.

7.

3x + 2≥7x − 13x + 2≥7x − 1

8.

−2x + 3> x − 5−2x + 3> x − 5

9.

4 (x + 3) ≥2x − 14 (x + 3) ≥2x − 1

10.

−12x≤ − 54 + 25x − 12x≤ − 54 + 25x

11.

−5 (x − 1) +3> 3x − 4−4x − 5 (x − 1) +3> 3x − 4−4x

12.

−3 (2x + 1)> — 2 (x + 4) −3 (2x + 1)> — 2 (x + 4)

13.

x + 38 − x + 55≥310x + 38 − x + 55≥310

14.

x − 13 + x + 25≤35x − 13 + x + 25≤35

Для следующих упражнений решите неравенство с абсолютным значением.Напишите окончательный ответ в виде интервалов.

19.

| х − 2 | + 4≥10 | х − 2 | + 4≥10

20.

| −2x + 7 | ≤13 | −2x + 7 | ≤13

22.

| х-20 |> -1 | х-20 |> -1

Для следующих упражнений опишите все значения x в пределах или включая расстояние от заданных значений.

24.

Дистанция 5 единиц от номера 7

25.

Расстояние 3 единицы от номера 9

26.

Дистанция 10 единиц от номера 4

27.

Расстояние 11 единиц от номера 1

Для следующих упражнений решите сложное неравенство. Выразите свой ответ, используя знаки неравенства, а затем запишите свой ответ, используя интервальную нотацию.

28.

−4 <3x + 2≤18−4 <3x + 2≤18

29.

3x + 1> 2x − 5> x − 73x + 1> 2x − 5> x − 7

30.

3y <5−2y <7 + y3y <5−2y <7 + y

31.

2x − 5 <−11or 5x + 1≥62x − 5 <−11or 5x + 1≥6

Графический

Постройте график функции для следующих упражнений.Наблюдая за точками пересечения, заштрихуйте ось x , представляющую решение, заданное для неравенства. Покажите свой график и напишите окончательный ответ в виде интервалов.

Для следующих упражнений нарисуйте обе прямые линии (левая сторона — y1, а правая сторона — y2) на одних и тех же осях. Найдите точку пересечения и решите неравенство, наблюдая, где оно истинно, сравнивая значения y линий.

41.

12x + 1> 12x − 512x + 1> 12x − 5

Числовой

Для следующих упражнений запишите набор в интервальной записи.

46. ​​

{x | xis все действительные числа} {x | xis все действительные числа}

Для следующих упражнений запишите интервал в нотации конструктора множеств.

50.

[−4,1] ∪ [9, ∞) [- 4,1] ∪ [9, ∞)

Для следующих упражнений запишите набор чисел, представленных в числовой строке, в интервальной записи.

52.

Технологии

Для следующих упражнений введите левую часть неравенства в виде графика Y1 в графической утилите.Введите y2 = правую часть. Ввод абсолютного значения выражения находится в меню MATH, Num, 1: abs (. Найдите точки пересечения, вспомните (2 nd CALC 5: пересечение, 1 st curve, enter, 2 nd ) кривой, введите, угадайте, введите). Скопируйте эскиз графика и заштрихуйте ось x для вашего решения, установленного на неравенство. Запишите окончательные ответы в виде интервалов.

Расширения

59.

Решите | 3x + 1 | = | 2x + 3 || 3x + 1 | = | 2x + 3 |

61.

x − 5x + 7≤0, x − 5x + 7≤0, x ≠ −7x ≠ −7

62.

p = −x2 + 130x − 3000p = −x2 + 130x − 3000 — формула прибыли для малого бизнеса. Найдите набор значений x , которые сохранят эту прибыль положительной.

Реальные приложения

63.

В химии объем для определенного газа определяется выражением V = 20T, V = 20T, где V измеряется в кубических сантиметрах, а T — температура в ºC. Если температура колеблется от 80ºC до 120ºC, найдите набор значений объема.

64.

Базовый пакет сотовой связи стоит 20 долларов в месяц. в течение 60 минут звонка с дополнительной оплатой в размере 0,30 доллара за минуту по истечении этого времени .. Формула стоимости будет следующей: C = 20 + 0,30 (x − 60) .C = 20 + 0,30 (x − 60). Если вам необходимо выставить счет на сумму не более 50 долларов, какое максимальное количество минут вы можете использовать?

Решите абсолютные неравенства — промежуточная алгебра

Цели обучения

К концу этого раздела вы сможете:

  • Решите уравнения абсолютных значений
  • Решите неравенства абсолютных значений с «меньше чем»
  • Решите неравенства абсолютных значений с помощью «больше чем»
  • Решить приложения с абсолютным значением

Прежде чем начать, пройдите тест на готовность.

  1. Оценить:

    Если вы пропустили эту проблему, просмотрите (рисунок).

  2. Введите или для каждой из следующих пар чисел.

    ⓐ ⓑ ⓒ ⓓ

    Если вы пропустили эту проблему, просмотрите (рисунок).

  3. Упростить:

    Если вы пропустили эту проблему, просмотрите (рисунок).

Решите уравнения абсолютных значений

Готовясь к решению уравнений абсолютного значения, мы пересматриваем наше определение абсолютного значения.

Абсолютное значение

Абсолютное значение числа — это его расстояние от нуля на числовой прямой.

Абсолютное значение числа n записывается как и для всех чисел.

Абсолютные значения всегда больше или равны нулю.

Мы узнали, что число и его противоположность находятся на одинаковом расстоянии от нуля на числовой прямой. Поскольку они находятся на одинаковом расстоянии от нуля, они имеют одинаковое абсолютное значение. Например:

на 5 единиц от 0, поэтому

на 5 единиц от 0, поэтому

(рисунок) иллюстрирует эту идею.

Цифры 5 и оба на пять единиц от нуля.

Для уравнения мы ищем все числа, которые подтверждают это утверждение. Мы ищем числа, расстояние от которых до нуля равно 5. Мы только что видели, что и 5, и пять единиц от нуля на числовой прямой. Они являются решениями уравнения.

Решение можно упростить до одного оператора, написав Это читается как « x равно положительному или отрицательному 5».

Мы можем обобщить это на следующее свойство для уравнений абсолютного значения.

Уравнения абсолютных значений

Для любого алгебраического выражения u и любого положительного действительного числа a ,

Помните, что абсолютное значение не может быть отрицательным числом.

Поскольку абсолютное значение всегда положительно, у этого уравнения нет решений.

Оба уравнения говорят нам об этом, и поэтому существует только одно решение.

ⓐⓑ нет решения ⓒ 0

ⓐⓑ нет решения ⓒ 0

Чтобы решить уравнение абсолютного значения, мы сначала выделяем выражение абсолютного значения, используя те же процедуры, которые мы использовали для решения линейных уравнений. Выделив выражение абсолютного значения, мы перепишем его как два эквивалентных уравнения.

Как решать уравнения абсолютных значений

Решить

Решить:

Решить:

Здесь приведены шаги для решения уравнения абсолютного значения.

Решите уравнения абсолютных значений.

  1. Выделите выражение абсолютного значения.
  2. Напишите эквивалентные уравнения.
  3. Решите каждое уравнение.
  4. Проверьте каждое решение.

Решить

Решить:

Решить:

Помните, абсолютное значение всегда положительно!

Решить:

Решить:

Решить:

Некоторые из наших уравнений абсолютных значений могут иметь форму, где u и v представляют собой алгебраические выражения.Например,

Как бы мы их разрешили? Если два алгебраических выражения равны по модулю, то они либо равны друг другу, либо отрицательны. Свойство для уравнений абсолютного значения говорит, что для любого алгебраического выражения u и положительного действительного числа a , if then или

Это говорит нам, что

Это приводит нас к следующему свойству для уравнений с двумя абсолютными значениями.

Уравнения с двумя абсолютными значениями

Для любых алгебраических выражений: u и v ,

Когда мы берем величину, противоположную количеству, мы должны быть осторожны со знаками и добавлять скобки там, где это необходимо.

Решение приложений с абсолютным значением

Неравенство абсолютных значений часто используется в производственном процессе. Изделие должно быть изготовлено с почти идеальными характеристиками. Обычно существует определенный допуск на отклонений от допустимых технических характеристик. Если отличие от спецификаций превышает допуск, товар отклоняется.

Идеальный диаметр стержня, необходимого для станка, — 60 мм. Фактический диаметр может отличаться от идеального на миллиметр.Какой диапазон диаметров будет приемлем для заказчика, не вызывая брака прутка?

Идеальный диаметр стержня, необходимого для станка, составляет 80 мм. Фактический диаметр может отличаться от идеального на 0,009 мм. Какой диапазон диаметров будет приемлем для заказчика, не вызывая брака прутка?

Диаметр стержня может составлять от 79,991 до 80,009 мм.

Идеальный диаметр стержня, необходимого для станка, — 75 мм.Фактический диаметр может отличаться от идеального на 0,05 мм. Какой диапазон диаметров будет приемлем для заказчика, не вызывая брака прутка?

Диаметр стержня может составлять от 74,95 до 75,05 мм.

Ключевые понятия

  • Абсолютное значение

    Абсолютное значение числа — это расстояние от 0 на числовой прямой.

    Абсолютное значение числа n записывается как и для всех чисел.

    Абсолютные значения всегда больше или равны нулю.

  • Уравнения абсолютных значений

    Для любого алгебраического выражения u и любого положительного действительного числа a ,

    Помните, что абсолютное значение не может быть отрицательным числом.

  • Как решать уравнения абсолютных значений
    1. Выделите выражение абсолютного значения.
    2. Напишите эквивалентные уравнения.
    3. Решите каждое уравнение.
    4. Проверьте каждое решение.
  • Уравнения с двумя абсолютными значениями

    Для любых алгебраических выражений u и v ,

  • Абсолютное неравенство значений с или

    Для любого алгебраического выражения u и любого положительного действительного числа a ,

  • Как решить неравенства абсолютных значений с помощью или
    1. Выделите выражение абсолютного значения.
    2. Напишите эквивалентное сложное неравенство.

    3. Решите составное неравенство.
    4. Постройте график решения
    5. Запишите решение, используя обозначение интервала
  • Абсолютное неравенство значений с или

    Для любого алгебраического выражения u и любого положительного действительного числа a ,

  • Как решить неравенства абсолютных значений с помощью или
    1. Выделите выражение абсолютного значения.
    2. Напишите эквивалентное сложное неравенство.

    3. Решите составное неравенство.
    4. Постройте график решения
    5. Запишите решение, используя обозначение интервала

Упражнения по разделам

Практика ведет к совершенству

Решите уравнения абсолютных значений

В следующих упражнениях решите.

ⓐⓑ нет решения ⓒ

ⓐⓑ нет решения ⓒ

Решите неравенства абсолютного значения с «меньше»

В следующих упражнениях решите каждое неравенство.Постройте график решения и запишите решение в интервальной записи.

Решите неравенства абсолютного значения с «больше чем»

В следующих упражнениях решите каждое неравенство. Постройте график решения и запишите решение в интервальной записи.

В следующих упражнениях решите. Для каждого неравенства также нанесите график решения и запишите решение в интервальной записи.

Решение приложений с абсолютным значением

В следующих упражнениях решите.

В идеале птицеферма производит 200 000 яиц в день. Но это общее количество может варьироваться на 25 000 яиц. Какая максимальная и минимальная ожидаемая продукция на ферме?

Завод по розливу органических соков в идеале производит 215 000 бутылок в день. Но эта сумма может варьироваться на 7 500 бутылок. Какое максимальное и минимальное ожидаемое производство на предприятии по розливу?

Ожидаемое производство от минимального до максимального — от 207 500 до 2225 000 бутылок

Чтобы гарантировать соблюдение закона, Мигель обычно превышает вес своих лепешек на 0.5 грамм. Он только что получил отчет, в котором говорилось, что он может терять до 100 000 фунтов стерлингов в год, используя эту практику. Теперь он планирует купить новое оборудование, которое гарантирует толщину лепешки в пределах 0,005 дюйма. Если идеальная толщина лепешки составляет 0,04 дюйма, то какая толщина лепешек будет гарантирована?

В пекарне Lilly’s идеальный вес буханки хлеба составляет 24 унции. По закону реальный вес может отличаться от идеального на 1,5 унции. Какой вес будет приемлемым для инспектора без наложения штрафа на пекарню?

Допустимый вес 22.От 5 до 25,5 унций.

Письменные упражнения

Напишите графическое описание абсолютного значения числа.

Объясните своими словами, как решить неравенство абсолютных значений,

Самопроверка

ⓐ После выполнения упражнений используйте этот контрольный список, чтобы оценить свое мастерство в достижении целей этого раздела.

ⓑ Что этот контрольный список говорит вам о вашем мастерстве в этом разделе? Какие шаги вы предпримете для улучшения?

Упражнения для повторения главы

Использование общей стратегии для решения линейных уравнений

Решение уравнений с использованием общей стратегии решения линейных уравнений

В следующих упражнениях определите, является ли каждое число решением уравнения.

В следующих упражнениях решите каждое линейное уравнение.

Классификация уравнений

В следующих упражнениях классифицируйте каждое уравнение как условное уравнение, тождество или противоречие, а затем сформулируйте решение.

противоречие; нет решения

Решение уравнений с дробными или десятичными коэффициентами

В следующих упражнениях решите каждое уравнение.

Используйте стратегию решения проблем

Используйте стратегию решения проблем для Word

В следующих упражнениях решайте, используя стратегию решения задач для текстовых задач.

Три четверти присутствующих на концерте — дети. Если на концерте 87 детей, сколько всего человек на концерте?

В группе девять саксофонистов. Количество саксофонистов на один меньше, чем в два раза больше, чем тубистов. Найдите количество игроков на тубе.

Решение проблем с числовыми словами

В следующих упражнениях решите каждую задачу с числовыми словами.

Сумма числа и тройки равна сорок одному.Найдите номер.

Одно число на девять меньше другого. Их сумма отрицательная двадцать семь. Найдите числа.

Одно число на два больше, чем в четыре раза больше. Их сумма — минус тринадцать. Найдите числа.

Сумма двух последовательных целых чисел — это Найти числа.

Найдите три последовательных четных целых числа, сумма которых равна 234.

Найдите три последовательных нечетных целых числа, сумма которых равна 51.

У Кодзи на сберегательном счету 5 502 евро.Это на 30 фунтов меньше, чем в шесть раз больше суммы на его текущем счете. Сколько денег у Кодзи на его текущем счете?

Процент решения

В следующих упражнениях переведите и решите.

Какое число составляет 67% от 250?

12,5% от какого числа 20?

Какой процент от 125 составляет 150?

В следующих упражнениях решите.

Счет за обед Дино составил 19,45 фунтов стерлингов. Он хотел оставить 20% от суммы счета в качестве чаевых.Сколько должны быть чаевые?

Долорес купила детскую кроватку на распродаже за 350 фунтов стерлингов. Цена продажи составила 40% от первоначальной цены. Какова была первоначальная цена детской кроватки?

Джейден зарабатывает 2680 фунтов стерлингов в месяц. Он платит 938 евро в месяц за аренду. Какой процент от его ежемесячной зарплаты идет на аренду?

Ангел получил повышение годовой зарплаты с 55 400 до 56 785 фунтов стерлингов. Найдите изменение в процентах.

Ежемесячный счет на бензин

Rowena снизился с 83,75 фунтов стерлингов в прошлом месяце до 56 фунтов стерлингов.95 в этом месяце. Найдите изменение в процентах.

Эммет купил пару туфель на распродаже со скидкой 40% от первоначальной цены 138 фунтов стерлингов. Найдите ⓐ сумму скидки и ⓑ продажную цену.

Лейси купила пару ботинок на распродаже за 95 фунтов стерлингов. Первоначальная цена ботинок составляла 200 фунтов стерлингов. Найдите ⓐ сумму скидки и ⓑ ставку дисконтирования. (При необходимости округлите до ближайшей десятой процента.)

ⓐ? 105

Нга и Лорен купили сундук на блошином рынке за 50 фунтов стерлингов.Они переработали его, а затем добавили наценку 350%. Найдите размер наценки и ⓑ прейскурантную цену.

Решение простых процентных заявок

В следующих упражнениях решите.

Уинстон положил 3294 евро на банковский счет с процентной ставкой 2,6% Сколько процентов было заработано за пять лет?

Мойра заняла у деда 4500 фунтов стерлингов, чтобы оплатить первый год обучения в колледже. Три года спустя она выплатила 4500 фунтов стерлингов плюс 243 фунта стерлингов.Какая была процентная ставка?

В выписке по кредиту на холодильник Хайме сказано, что он заплатит 1026 фунтов стерлингов в виде процентов за четырехлетний кредит под 13,5%. Сколько Хайме занял, чтобы купить холодильник?

Решите формулу для конкретной переменной

Решение формулы для конкретной переменной

В следующих упражнениях решите формулу для указанной переменной.

Решите формулу

для L .

Решите формулу

на т .

Решите формулу

для у .

Использование формул для решения геометрических задач

В следующих упражнениях решите, используя геометрическую формулу.

Какова высота треугольника с площадью квадратных метров и основанием 9 метров?

Размер наименьшего угла в прямоугольном треугольнике меньше, чем размер следующего большего угла. Найдите размеры всех трех углов.

Периметр треугольника 97 футов.Одна сторона треугольника на одиннадцать футов больше самой маленькой стороны. Третья сторона на шесть футов больше, чем вдвое меньшая сторона. Найдите длины всех сторон.

Найдите длину гипотенузы.

Найдите длину отсутствующей стороны. При необходимости округлите до ближайшей десятой.

Серджио нужно прикрепить провод, чтобы прикрепить антенну к крыше своего дома, как показано на рисунке. Антенна восьми футов высотой, а у Серджио 10 футов провода.Как далеко от основания антенны можно прикрепить провод? При необходимости с точностью до десятых долей.

Сеонг строит стеллажи в своем гараже. Полки имеют ширину 36 дюймов и высоту 15 дюймов. Он хочет прикрепить диагональную скобу к спине, чтобы стабилизировать полки, как показано. Какой длины должна быть скобка?

Длина прямоугольника на 12 см больше ширины. Периметр 74 см. Найдите длину и ширину.

см, см

Ширина прямоугольника в три раза больше длины, чем в два раза.По периметру 96 дюймов. Найдите длину и ширину.

Периметр треугольника составляет 35 футов. Одна сторона треугольника на пять футов длиннее второй. Третья сторона на три фута длиннее второй. Найдите длину каждой стороны.

Приложения для решения задач смешивания и равномерного движения

Решите проблемы с монетным словом

В следующих упражнениях решите.

У Полетт есть купюры номиналом 5 и 10 евро по 140 евро. Количество купюр? 10 на один меньше, чем вдвое, количества купюр? 5.Сколько у нее каждого?

У Ленни 3,69 фунта в пенни, десять центов и четвертаки. Количество пенни на три больше, чем количество монет. Количество четвертей в два раза больше, чем десятицентовик. Сколько у него каждой монеты?

девять пенсов, шесть центов, 12 четвертей

Решение проблем со словами о билетах и ​​штампах

В следующих упражнениях решите каждую проблему со словом билета или штампа.

Билеты на баскетбольный матч стоят 2 евро для студентов и 5 евро для взрослых.Количество студентов было на три меньше, чем количество взрослых, более чем в 10 раз. Общая сумма денег от продажи билетов составила 619 евро. Сколько было продано каждого билета?

На концерт джаз-бэнда было продано

125 билетов на общую сумму 1022 евро. Студенческие билеты стоят 6 евро каждый, а общие входные билеты — 10 евро. Сколько билетов каждого вида было продано?

Юми потратила 34,15 евро на покупку марок. Количество купленных марок по 0,56 евро было на 10 меньше, чем в четыре раза больше, чем количество фунтов стерлингов.41 марка. Сколько штук каждого она купила?

Решить проблемы со смешанными словами

В следующих упражнениях решите.

Marquese делает 10 фунтов сухой смеси из изюма и орехов. Изюм стоит 3,45 фунтов за фунт, а орехи — 7,95 фунтов за фунт. Сколько фунтов изюма и сколько фунтов орехов должен использовать Маркез для приготовления смеси для троп, чтобы стоить ему 6,96 фунтов за фунт?

фунта изюма, фунта орехов

Эмбер хочет положить плитку на кухонную плиту.Ей понадобится 36 квадратных футов плитки. Она будет использовать базовые плитки, которые стоят 8 фунтов за квадратный фут, и декоративные плитки, которые стоят 20 фунтов за квадратный фут. Сколько квадратных футов каждой плитки она должна использовать, чтобы общая стоимость фартука составила 10 фунтов за квадратный фут?

Энрике взял взаймы 23 500 евро на покупку автомобиля. Он платит своему дяде 2% годовых на 4500 фунтов стерлингов, которые он взял у него в долг, а на оставшуюся часть он платит банку 11,5% годовых. Какую среднюю процентную ставку он платит на общую сумму 23 500? (Округлите ответ до ближайшей десятой процента.)

 *** QuickLaTeX не может составить формулу:
9.7 \ текст {%}

*** Сообщение об ошибке:
Файл завершился при сканировании использования \ text @.
Экстренная остановка.

 

Решение Uniform Motion Applications

В следующих упражнениях решите.

Гейб едет из Сакраменто в Реддинг, это занимает у него 2,2 часа. Эльза преодолевает такое же расстояние за два часа. Скорость Эльзы на семь миль в час выше скорости Гейба. Найдите скорость Гейба и скорость Эльзы.

Луэллен и Трейси встретились в ресторане на дороге между Чикаго и Нэшвиллом. Луэллен покинул Чикаго и ехал 3,2 часа в сторону Нэшвилла. Трейси покинула Нэшвилл и ехала 4 часа в сторону Чикаго со скоростью на одну милю в час быстрее, чем скорость Луэллена. Расстояние между Чикаго и Нэшвиллом составляет 472 мили. Найдите скорость Луэллен и скорость Трейси.

Louellen 65 миль / ч, Трейси 66 миль / ч

Два автобуса отправляются из Амарилло одновременно. Автобус из Альбукерке направляется на запад по I-40 со скоростью 72 мили в час, а автобус из Оклахома-Сити направляется на восток по I-40 со скоростью 78 миль в час.Сколько часов им понадобится, чтобы разойтись на 375 миль?

Кайл греб на своей лодке против течения 50 минут. Ему потребовалось 30 минут, чтобы плыть обратно вниз по течению. Его скорость вверх по течению на две мили в час меньше, чем скорость вниз по течению. Найдите скорость Кайла вверх и вниз по потоку.

до 3 миль в час, ниже по потоку 5 миль в час

В 6:30 Девон вышла из дома и ехала на велосипеде по ровной дороге до 7:30. Потом она начала кататься в гору и ехала до 8:00. Всего она проехала 15 миль.Ее скорость на ровной дороге была на три мили в час выше, чем при подъеме в гору. Найдите скорость Девон на ровной дороге и в гору.

Энтони ехал из Нью-Йорка в Балтимор, что составляет 192 мили. Он уехал в 3:45, и движение было загружено до 5:30. Движение было слабым, и он прибыл в 7:30. Его скорость в легком транспортном потоке была на четыре мили в час больше, чем в два раза его скорость в плотном потоке. Узнайте скорость движения Энтони в условиях интенсивного и легкого движения.

интенсивное движение 32 миль / ч, легкое движение 66 миль / ч

Решите линейные неравенства

Неравенства в графике на числовой прямой

В следующих упражнениях нарисуйте неравенство на числовой прямой и запишите в интервальной записи.

Решите линейные неравенства

В следующих упражнениях решите каждое неравенство, нанесите решение на числовой прямой и запишите решение в интервальной записи.

Перевести слова в неравенство и решить

В следующих упражнениях переведите и решите. Затем запишите решение в интервальной записи и нанесите график на числовой прямой.

Пять больше z не больше 19.

Три менее c не менее 360.

Дважды отрицательное число, умноженное на , — не более восьми.

Решение приложений с линейными неравенствами

В следующих упражнениях решите.

Еженедельный бюджет Джулианны на питание для ее семьи составляет 231 евро. Если она планирует выделять одинаковую сумму на каждый из семи дней недели, какова максимальная сумма, которую она может тратить на еду каждый день?

Рохелио пишет акварелью. Он получил подарочную карту на 100 фунтов стерлингов в магазин товаров для искусства и хочет использовать ее, чтобы купить холсты размером 12 дюймов на 16 дюймов. Каждый холст стоит 10,99 евро.Какое максимальное количество холстов он может купить по подарочной карте?

Бриане предложили работу продавца в другом городе. Предложение было за 42 500 фунтов стерлингов плюс 8% от общего объема продаж. Чтобы переезд окупился, Бриана должна иметь годовую зарплату не менее 66 500 фунтов стерлингов. Какой должна быть сумма ее продаж, чтобы она переехала?

Автомобиль Рене обходится ей в 195 фунтов в месяц плюс 0,09 фунтов за милю. Сколько миль может проехать Рене, чтобы ее ежемесячные расходы на машину не превышали 250 фунтов стерлингов?

Коста — бухгалтер.Во время налогового сезона он взимает 125 евро за простую налоговую декларацию. Его расходы на покупку программного обеспечения, аренду офиса и рекламу составляют 6000 фунтов стерлингов. Сколько налоговых деклараций он должен сделать, если он хочет получить прибыль не менее 8000?

Дженна планирует пятидневный курортный отпуск с тремя своими друзьями. Это будет стоить ей 279 фунтов стерлингов за авиабилеты, 300 фунтов стерлингов за еду и развлечения и 65 фунтов стерлингов в день за ее долю в отеле. Она накопила 550 фунтов стерлингов на свой отпуск и может зарабатывать 25 фунтов в час в качестве ассистента в фотостудии своего дяди.Сколько часов она должна работать, чтобы иметь достаточно денег на отпуск?

Решите сложные неравенства

Решите сложные неравенства с помощью «и»

В каждом из следующих упражнений решите каждое неравенство, нанесите решение на график и запишите решение в интервальной записи.

и

и

Решите сложные неравенства с помощью «или»

В следующих упражнениях решите каждое неравенство, нанесите решение на числовой прямой и запишите решение в интервальной записи.

или

или

Решение приложений со сложными неравенствами

В следующих упражнениях решите.

Лиам играет в числа со своей сестрой Одри. Лиам придумывает число и хочет, чтобы Одри его угадала. Число от 2 до 32 в пять раз больше, чем в три раза. Напишите составное неравенство, показывающее диапазон чисел, о котором может думать Лиам.

Элуиза создает прямоугольный сад на заднем дворе.Длина сада 12 футов. Периметр сада должен быть не менее 36 футов и не более 48 футов. Используйте составное неравенство, чтобы найти диапазон значений ширины сада.

Решите абсолютные неравенства значений

Решите уравнения абсолютных значений

В следующих упражнениях решите.

Решите неравенства абсолютного значения с «меньше»

В следующих упражнениях решите каждое неравенство.Постройте график решения и запишите решение в интервальной записи.

Решите неравенства абсолютного значения с «больше чем»

В следующих упражнениях решите. Постройте график решения и запишите решение в интервальной записи.

Решение приложений с абсолютным значением

В следующих упражнениях решите.

Производителю крафтового пива требуется 215 000 бутылок в день. Но эта сумма может варьироваться на 5 000 бутылок. Какое максимальное и минимальное ожидаемое использование в компании по розливу?

Ожидаемое минимальное и максимальное использование — от 210 000 до 220 000 бутылок

В магазине Fancy Grocery идеальный вес буханки хлеба составляет 16 унций. По закону реальный вес может отличаться от идеального на 1,5 унции. Какой вес будет приемлемым для инспектора без наложения штрафа на пекарню?

Практический тест

В следующих упражнениях решите каждое уравнение.

противоречие; нет решения

Решите формулу

для у .

В следующих упражнениях нарисуйте неравенство на числовой прямой и запишите в интервальной записи.

В следующих упражнениях решите каждое неравенство, нанесите решение на числовой прямой и запишите решение в интервальной записи.

и

или

В следующих упражнениях переведите уравнение или неравенство и решите.

Четыре меньше, чем в два раза x равно 16.

Найдите длину отсутствующей стороны.

Одно число на четыре больше, чем в два раза больше другого. Их сумма — найти числа.

Сумма двух подряд идущих нечетных чисел — это Найти числа.

Маркус купил телевизор на распродаже за 626,50 фунтов стерлингов. Первоначальная цена телевизора составляла 895 фунтов стерлингов. Найдите ⓐ сумму скидки и ⓑ ставку дисконтирования.

У Бониты в кармане 2,95 евро в десятицентовых монетах. Если у нее на пять центов больше, чем четвертей, сколько каждой монеты у нее?

Ким делает восемь галлонов пунша из фруктового сока и содовой. Фруктовый сок стоит 6,04 фунта за галлон, а газировка стоит 4,28 фунта за галлон. Сколько фруктового сока и сколько соды нужно употребить, чтобы пунш стоил? 5.71 на галлон?

Размер одного угла треугольника в два раза больше размера наименьшего угла. Мера третьего угла в три раза больше меры наименьшего угла. Найдите размеры всех трех углов.

Длина прямоугольника на пять футов больше ширины в четыре раза. Периметр — 60 футов. Найдите размеры прямоугольника.

Два самолета вылетают из Далласа одновременно. Один направляется на восток со скоростью 428 миль в час.Другой самолет летит на запад со скоростью 382 мили в час. Сколько часов потребуется, чтобы их разделять 2025 миль?

часов

Леон поехал из своего дома в Цинциннати в дом своей сестры в Кливленде, на расстоянии 252 мили. На это у него ушло несколько часов. Первые полчаса у него было интенсивное движение, а в остальное время его скорость была на пять миль в час меньше, чем в два раза его скорость в условиях интенсивного движения. Какая была его скорость в условиях интенсивного движения?

У Сары есть бюджет в 1000 фунтов стерлингов на костюмы для 18 членов ее музыкальной театральной группы.Какую максимальную сумму она может потратить на каждый костюм?

Максимум? 55,56 за костюм.

Абсолютные неравенства

Неравенства абсолютных значений

Помните, абсолютное значение означает расстояние от нуля на числовой прямой. | x | <4 означает, что x — это число, которое меньше 4 единиц от нуля на числовой прямой (см. Рисунок 1).

Рисунок 1. Меньше 4 от нуля.

Решения — это числа справа от –4 и слева от 4 и могут быть обозначены как

.

| x | > 4 означает, что x — это число, которое больше чем на 4 единицы от нуля на числовой прямой (см. Рисунок 2).

Рисунок 2. Более 4 из 0.

Решения — это числа слева от –4 или справа от 4 и обозначаются как

.

{ x | x <–4 или x > 4}

| x | <0 не имеет решений, тогда как | x | > 0 имеет своим решением все действительные числа, кроме 0.| x | > –1 имеет своим решением все действительные числа, потому что после взятия абсолютного значения любого числа этот ответ будет либо нулем, либо положительным и всегда будет больше, чем –1.

Ниже приводится общий подход к решению неравенств по абсолютным значениям вида

Пример 1

Решить для x : | 3 x — 5 | <12.

Набор решений —

График набора решений показан на рисунке 3.

Рис. 3. x больше и меньше.

Пример 2

Решите эту дизъюнкцию для x : | 5 x + 3 | > 2.

Набор решений есть. График набора решений показан на рисунке 4.

Рис. 4. x меньше –1 или больше.

Пример 3

Решить относительно x : | 2 x + 11 | <0.

Это неравенство не имеет решения.

Пример 4

Решить относительно x : | 2 x + 11 | > 0,

Решением являются все действительные числа , кроме для решения 2 x + 11 = 0. Следовательно,

Решение набора есть. График набора решений показан на рисунке 5.

Рис. 5. Все числа, кроме.

Пример 5

Решить относительно x : 7 | 3 x + 2 | + 5> 4.

Сначала выделите нажатие e x , содержащее символ абсолютного значения.

Набор решений — это все действительные числа. ( Примечание: Абсолютное значение любого числа всегда равно нулю или положительному значению. Следовательно, абсолютное значение любого числа всегда больше, чем отрицательное значение.) График набора решений показан на рисунке 6.

Рисунок 6. Набор всех чисел.

Как найти и построить график абсолютного неравенства — математический класс [видео 2021 года]

Неравенство абсолютных значений с одной переменной

Итак, теперь, когда обзор закончился, давайте объединим эти два навыка в один настолько просто, насколько это возможно.Решить и построить график | x | > 5.

Не узнавая ничего нового, попробуем решить эту проблему так же, как и раньше. Если меня просят решить, и у меня есть абсолютное значение, мне нужно отменить его, разделив неравенство на части. Это оставляет мне два неравенства: x > 5 или x > -5.

Что ж, это было неплохо. Теперь у меня остается сложное неравенство, потому что у меня два неравенства в одном, но это нормально. Давайте проверим наши ответы, чтобы убедиться, что это действительно так просто.

x > 5 говорит, что числа типа 6 или 100 должны работать в исходной задаче, поэтому мы можем подставить их, взять абсолютные значения и, конечно же, проверить их.

Пример неравенства сложного и сложного

Хорошо, теперь x > -5. Это говорит о том, что числа вроде -4 или 1 должны работать. Таким образом, мы можем заменить их в исходной задаче, взять абсолютные значения и … да, это не так.Поскольку абсолютное значение делает их положительными, 4 и 1 не работают, потому что их величина не такая большая, как 5. Так что нам действительно нужны «большие» отрицательные числа, такие как -10 или -100, потому что тогда абсолютные ценность изменит их снова на положительную сторону, и мы будем в бизнесе. Таким образом, вместо x > -5 нам действительно нужно x <-5, потому что числа вроде -10 и -100 технически меньше -5, хотя они «большие» или более высокие числа.

То, что мы только что обнаружили, — это золотое правило решения неравенств абсолютных значений с одной переменной.При разделении неравенства на два новых, чтобы отменить абсолютное значение, нам нужно перевернуть знак того, который мы установили на отрицательный. Я лично помню это, думая об этом как об умножении и делении по отрицательному правилу, где вы должны перевернуть знак. Это вроде как.

В любом случае, теперь, когда у нас есть правильный ответ: x > 5 или x <-5, мы можем изобразить составное неравенство, используя наши предыдущие знания.

Выполняя одно неравенство за раз и ставя пустой кружок на 5, потому что это просто «больше чем», а не «или равно», и рисование стрелки вправо, чтобы указать все числа больше 5, покрывает половину график.Затем поместите еще один открытый кружок на -5 и проведите стрелку влево, чтобы указать, что меньшие числа позаботятся обо всем остальном.

Глядя на график, мы видим, что мы имеем сложное неравенство «или», потому что две стрелки направлены в противоположные стороны. Это означает, что числа больше 5 или числа меньше -5 будут работать в наших исходных неравенствах.

Неравенство по абсолютным значениям может также привести к сложному неравенству. Например: решить и построить график | x | <5

Теперь вместо поиска очень «больших» чисел, которые должны быть больше пяти после того, как они станут положительными, мы ищем выбранную группу «маленьких» чисел, которые все равно будут меньше пяти. даже после того, как они были возвращены в положительную сторону.

Использование нашего нового навыка переворачивания символа неравенства при отмене абсолютного значения дает нам решенное неравенство как x <5 и x > -5. Помните, мне пришлось его перевернуть.

Причина, по которой это неравенство «и» состоит в том, что для того, чтобы наше решение работало, оно должно удовлетворять обоим условиям. Оно не только должно быть меньше 5, но также должно быть больше -5. Это не сработает, если верно только одно из этих утверждений, а не другое.Возьмем, к примеру, 10: 10 больше -5, но не меньше 5. Поэтому, когда мы подставим 10 обратно в исходное неравенство, вы обнаружите, что это не работает. Любое составное неравенство, в котором должны выполняться оба условия, чтобы ваше решение работало, является составным неравенством «и». Как только мы построим график неравенства, мы получим это и увидим, что только «маленькие» числа от -5 до 5 будут по-прежнему меньше 5 после того, как будут изменены на положительное.

Неравенства с двумя переменными абсолютных значений

Наконец, мы также можем построить график неравенства с двумя переменными абсолютных значений, например: y <| x -2 | +1.

Как и другие графики неравенства с двумя переменными, нам нужно начать с построения граничной линии прямо там, где y = | x -2 | + 1.

Используйте навыки перевода, чтобы построить границу для уравнения

Это требует от нас использования наших навыков перевода: используйте -2 внутри абсолютного значения, чтобы сместить вершину вправо, и положительную единицу снаружи, чтобы переместить вершину на 1.Поскольку перед абсолютным значением нет переменной, мы можем предположить, что там есть небольшое воображаемое число 1, и это говорит нам, что наклон V равен 1, что означает, что от моей вершины я поднимаюсь на 1, а затем на 1. в обоих направлениях, чтобы найти V.

Теперь нам нужно решить, какая сторона буквы V является стороной «больше, чем», а какая — стороной «меньше». Подстановка исходной точки (0,0) в исходное неравенство по-прежнему является хорошей стратегией, и это дает нам 0 <| 0-2 | + 1. Если это упростить, то получим 0 <3, что является верным утверждением.Следовательно, область графика, в которой находится начало координат, является той частью графика, которую мы должны заштриховать. Это означает, что мы закрашиваем все снаружи V - слева, под ним и справа.

Наконец, поскольку неравенство было строго «меньше», а не «равно», мы делаем V пунктирной линией, чтобы указать, что оно не является частью решения. После того, как все шаги выполнены, мы закончили, и у нас есть этот график в качестве нашего ответа.

Пунктирная линия показывает, что это не часть решения.

Резюме урока

При решении неравенств по абсолютным значениям с одной переменной следует помнить одно новое правило: при отмене абсолютного значения путем разделения уравнения на два необходимо перевернуть символ неравенства в неравенстве, которое установлен на отрицательное значение (вроде как целое «переворачивание знака при умножении или делении на отрицательное»).

Неравенства с одной переменной по абсолютной величине дают нам сложных неравенств .

Абсолютное значение, превышающее что-либо, дает нам составное неравенство «или», тогда как абсолютное значение, которое меньше чего-либо, дает нам составное неравенство «и».

Построение графиков Неравенства с двумя переменными абсолютных значений В нет новых правил, которые нужно запомнить, и ваши ответы должны выглядеть как V, с заштрихованной внутренней или внешней стороной V.

Цели урока

По завершении этого урока вы сможете:

  • Решать неравенства абсолютных значений с одной и двумя переменными
  • График 1 и 2 неравенства абсолютных значений

Неравенства абсолютного значения

Выражения абсолютного значения не обязательно должны быть абсолютно равными; у них тоже может быть неравенство.Эээ, неравенства. Однако все становится немного странно, когда абсолютные ценности и неравенства находятся в одной комнате вместе.

Пример задачи

Решить | x | <1.

Если бы это было уравнение, у нас было бы следующее: x = 1 или x = -1. Но подумайте, что на самом деле означает абсолютное значение. Это числовое расстояние от 0. Если у нас есть | x | <1, нам нужны все числа, расстояние от которых до 0 меньше 1.

Ага, это они. На числовой строке мы видим, что x = -1 и x = 1 являются границами этого сложного неравенства, -1 < x <1. Другими словами, это x > -1 и . x <1. Иначе говоря, решения нашего неравенства лежат между ± 1. Надо ли говорить об этом по-четвертому? Надеюсь, что нет, потому что мы отключились.

Эта модель повторяется для многих неравенств по абсолютным значениям.Если у нас есть | (кое-что) | <какое-то число n , тогда наш ответ — n <(немного) < n . Затем мы решаем нашу переменную. Не , а , другой, внутри. Мы не показывали это здесь, но вы знаете, что это есть.

Пример задачи

Решить | 2 x + 3 | ≤ 5.

Частичное равенство знака ничего не меняет. Очень похоже на тот дешевый мотель, в котором мы останавливались во время поездки прошлым летом. Мы получаем нервозность, просто вспоминая простыни в этом месте.

У нас есть какой-то знак «меньше», поэтому любые решения, которые мы можем найти, будут между -5 и 5.

-5 ≤ 2 x + 3 ≤ 5

Да, вот так. Теперь мы можем найти нашу переменную. Начнем с вычитания 3 из каждого раздела.

-8 ≤ 2 x ≤ 2

Пора отделить коэффициент от переменной. Наши ладони начали потеть; Как насчет твоего? Должны, так как мы разделяем знаки неравенства вокруг. А 2 — положительный результат, так что все в порядке.Вытирание рук полотенцем и деление дает нам наши решения:

-4 ≤ x ≤ 1

Это все хорошо, но что мы будем делать, когда абсолютное значение «больше» числа? Ответ не будет таким же, как в случае «меньше чем». Это не может быть просто, правда?

Пример задачи

Решить | x | ≥ 6.

Давайте снова подумаем об этом с точки зрения расстояния: нам нужны все числа, которые имеют расстояние больше или равное 6 единицам от 0.Как только у нас будут все эти числа, мы наденем им платья и устроим чаепитие.

Похоже, наши решения могут быть меньше или равны -6 или больше или равны 6. Это еще одно сложное неравенство, но на этот раз к нему добавлено «или»:

x ≤ — 6 или x ≥ 6

Может показаться странным, что числа меньше -6 могут быть решением проблемы, которая начинается со знака «больше чем», но вставьте число и проверьте его: is | -7 | ≥ 6? Ага, | -7 | 7.И 7 определенно больше и страшнее 6, потому что 7 8 9.

Пример задачи

Решить | -2 x — 1 | > 7.

Первое, что мы делаем — встаем и потягиваемся. Мы долгое время работали над этим неравенством, и это убивает нас. Сделав это, мы проверяем знак неравенства. Это знак «больше чем», поэтому мы знаем, что у нас будет сложное «неравенство». Одно из неравенств будет нашим исходным выражением, только без полосок абсолютных значений.

-2 x — 1> 7

-2 x > 8

x <-4

Вы это видели? Отрицательное деление — надо поменять все знаки. Все они, включая знак неравенства.

Другая половина решения будет противоположной тому, что находится внутри столбиков.

— (- 2 x — 1)> 7

-2 x — 1 <-7

У нас больше отрицательного умножения. Отмените это неравенство, переверните его хорошо.

-2 x <-6

x > 3

И тогда мы должны снова изменить его. Шиш, уже решайся. Вы хотите указать влево или вправо?

Наше полное решение — x <-4 или x > 3. Мы немного нервничаем по поводу нашего ответа; там происходило много перебранных знаков, и мы слышали тревожные звуки, пока он был в другой комнате. Так что давайте еще раз проверим.

Подключение -5, что меньше -4:

| -2 (-5) — 1 | > 7

| 10 — 1 | > 7

| 9 | > 7

Подключение 4, которое больше 3:

| -2 (4) — 1 | > 7

| -8 — 1 | > 7

| -9 | > 7

Ладно, все круто.И оказывается, что эти звуки были просто расстроенной скрипкой. Мы думали, он пытался вызвать кучу пауков. Мы были , а эти были близки к тому, чтобы вызвать истребитель.

Резюме

Столкнулись с абсолютным неравенством значений и не знаете, что делать?

  • Если неравенство имеет знак «меньше» (<) с выражением абсолютного значения слева, то решение является составным »и« неравенством между ± константой ».
  • Если неравенство имеет знак «больше» (>) с выражением абсолютного значения слева, то решением является составное «или» неравенство.
    • Первая половина — это исходное выражение без знаков абсолютного значения.
    • Вторая половина переворачивает знак неравенства и меняет все, что находится за пределами столбцов абсолютного значения, на противоположный знак.
  • Если вы не можете вспомнить эти правила или просто не знаете, что делать, начертите небольшую числовую черту. Глядя на него, вы не напутаете.

Быстрый вопрос: что будет, если мы попытаемся найти | x | <-1? Или | x | > -1? Происходит следующее: «Это ловушка!» Абсолютное значение всегда будет положительным, поэтому решения нет.

Добавить комментарий

Ваш адрес email не будет опубликован. Обязательные поля помечены *

2024 © Все права защищены.